SlideShare a Scribd company logo
1 of 123
THORACIC
MALIGNANCY
• Prof.Dr.A.SundararajaperumalM.D;D.C.H
• Professor
• Institute Of Thoracic Medicine
• Madras Medical College & RGGGH
• Chennai - 3
Overview
Lung Cancer Screening
Fleischner’s Guidelines
Diagnostic workup of Lung Cancer
Staging of Lung Cancer
Complications
Case
Scenario 1
A 60 yrs old, Mr.G comes to our hospital with
chronic cough.
Chronic smoker, 30 pack years.
A farmer by occupation.
He is a known case of COPD/CAD/past history of
surgery for hydrocele.
He is currently on LAMA/LABA inhaler 2 puffs
daily, on dual antiplatelets.
He is other wise able to carry out all his activities
on his own.
His son wants to know if his father could develop
lung cancer and if there are any screening tests.
Question 1
• 1. Screening tests are not very effective in Lung
cancer
• 2. Screening test with low dose CT scan is an option
but this patient is not a suitable candidate.
• 3.Screening can be offered to him at 1,2,3.5yrs
respectively
• 4.Screening can be offered to him at 1,2,2.5 yrs
respectively.
Screening – Lung Malignancy
• Only recommended screening test for lung cancer is Low-dose computed
tomography (also called a low-dose CT scan, or LDCT).
• Who Should Be Screened?
 Have a 20 pack-year or more smoking history, and
 Smoke now or have quit within the past 15 years, and
 Are between 50 and 80 years old.
The National Lung Screening Trial (NLST)
• Evaluated annual LDCT vs CXR screening in 53,454 patients aged 55–74 y
with a 30-pack year smoking
• History (including those who quit within 15 y), showing a 20% reduction in
lung cancer and 6.7% reduction in all-cause mortality in the CT arm.
• 320 patients would need to be screened to prevent one lung cancer death.
NELSON RCT-EUROPE
• The NELSON European RCT included 15,822 current/former
smokers
• Evaluated LDCT screening at increasing screening intervals (1, 2,
and 2.5 years), compared with no screening.
• Presented data suggests male mortality reduction of 26% at 10 y,
female mortality reduction of 39% at 10 y
Pulmonary Nodules
• Pulmonary nodules are frequently encountered on chest CT screening
• Nodule characterization should be performed on thin-slice CT images
≤1.5 mm, since a small solid nodule may appear to have groundglass
density on a thick slice due to partial-volume effect.
Pulmonary Nodule
Pulmonary nodules can be divided into solid
lesions and subsolid lesions, which can be
further subdivided into part-solid and pure
ground glass nodules.
Subsolid nodule (SSN)
A pulmonary nodule with at least partial
groundglass appearance
Groundglass
Opacification with a higher density than the
surrounding tissue, not obscuring underlying
bronchovascular structures
Subsolid nodules
• Most subsolid nodules are transient and the result of infection or
hemorrhage.
• However, persistent subsolid nodules often represent pathology in the
adenocarcinomatous spectrum.
• No reliable distinction can be made radiologically, although studies suggest
that larger size and a solid component are associated with more invasive
behaviour.
• Compared to solid lesions, persistent subsolid nodules have a much slower
growth rate, but carry a much higher risk of malignancy.
• In a study by Henschke et al., part-solid nodules were malignant in 63%,
pure groundglass SSNs in 18% and solid nodules only in 7%
Subsolid nodules
Solid nodules
Solid pulmonary nodules can represent various etiologies
• Benign granulomas
• Focal scar
• Intrapulmonary lymph nodes
• Primary malignancies
• Metastatic disease.
Solid nodules
Probability of Malignancy
Low ( ˂5% ) Intermediate
( 5-65% )
High ( ˃65% )
• Young
• Less smoking
• No Prior Cancer
• Small nodule size
• Regular margin
• Non-Upper Lobe
• Mixture of Low and
High probability
features
• Older
• Heavy Smoking
• Prior Cancer
• Larger size
• Irregular Margin
• Upper lobe location
• Emphysema
• Pulmonary fibrosis ( IPF )
• Family H/O Lung cancer
Fleischner guideline for Pulmonary nodule
management
• Only apply in subjects 35 years or older
• Do not apply in immunocompromised
subjects or pts with a known or
suspected malignancy
• Use Thin-slice CT imaging ( LDCT )
• Manual nodule measurements should be
based on the avg of long and short axis
diameters. Alternatively use nodule
volumetry.
• Risk - stratification on both patient and
nodule charectristics
Perifissural nodule
• Perifissural nodules are a separate and benign
entity.
• A typical PFN is attached to a pulmonary fissure, is
homogeneous, and solid with smooth margins. The
shape is oval, lentiform or triangular
• A nodule with these specific characteristics needs
no follow-up and is probably an intrapulmonary
lymph node.
• In a study by de Hoop none of the 919 typical and
atypical PFNs were found to be malignant in 5.5
year follow-up.
• Typical PFNs can show significant growth rates on
serial imaging comparable to malignant nodules.
• This is not a sign of malignancy, but merely a result
of their lymphatic origin.
CT Chest of the patient
When Should Screening Stop?
 Turns 81 years old, or
 Has not smoked in 15 or more years, or
 Develops a health problem that makes him or her
unwilling or unable to have surgery if lung cancer
is found.
Biomarker screening
• Cancer antigen 125
• Carcinoembryonic antigen
• Cytokeratin-19 fragment
• The precursor of surfactant protein B
In the validation study of 63 ever-smoking patients with lung cancer and 90
matched controls, an integrated risk prediction model that combined
smoking exposure with the biomarker panel score identified 40 of the 63
lung cancer cases, corresponding to a sensitivity of 0.63. By comparison, the
US Preventive Services Task Force screening criteria demonstrated a
sensitivity of 0.42 for these cases.
Case Scenario 2
• 56 yrs old male came for master health check up, his chest X ray
revealed right lung solitary pulmonary nodule
• Patient has no symptoms
• Not a Smoker
• Systemic examination – no significant abnormality
Case Scenario 2
• Solitary pulmonary nodule in
chest X- ray.
• Whether to observe or
investigate further?
Case Scenario 2
• Next plan of action to this patient?
• Discharge the patient and ask him not
to come unless and otherwise there are
symptoms
• Do a whole-body PET
• Ask the radiologist the size of the
nodule and decide further course of
action
• Refer the patient to the surgeon for
lobectomy
CT Chest
• A 30 x 27 mm sized nodular mass lesion in the right middle lobe of 35 –
45 HU CT density in plain study and shows lobulated contour with
multiple thin subpleural strips.
• It shows few air bronchogram and feeding pulmonary vessel sign
surrounded with focal areas of ground glass attenuation
• 12 x 6 mm right paratracheal lymphnode and 4-8 mm sized aorto –
pulmonary lymphnode
Solitary pulmonary nodule malignancy risk in adults
Herder Calculator
Brock Calculator
Answers calculated to formulate result:
1. Age? — 56 Years
2. Smoker (current or previous)? — No
3. Extra-thoracic cancer more than 5 years previous? — No
4. Diameter? — 30 mm
5. Upper Lobe? — No
6. Spiculated? — Yes
7. PET? — Moderate
Results for Solitary Pulmonary Nodule Malignancy Risk
Probability of Malignancy: 87.3 %
Algorithm for nodule
follow up
Nodule Charecteristics
• 30mm
• Brock Model risk score 36.92%
• Herder Model risk 87.3%
• As per the algorithm the patient needs biopsy for finding out the
definitive diagnosis
• What is the diagnostic method of choice in this case?
What is the most appropriate next step
1.Bronchoscopy
2.CT guided biopsy
3.Do a PET scan and decide
4.EBUS
HPE image of the Biopsy
• composed of goblet or columnar cells with abundant mucin
Non-Small cell
Lung Cancer
• Non–small cell lung cancer (NSCLC) accounts for
approximately 85% of all lung cancers
• Histologically, NSCLC is divided into
• adenocarcinoma,
• squamous cell carcinoma (SCC)
• large cell carcinoma
• Often insidious, producing no symptoms
• At initial diagnosis,
• 20% of patients have localized disease
• 25% of patients have regional metastasis
• 55% of patients have distant spread of disease
Etiology
Exposure to carcinogens
• Tobacco smoke contains more than 300 harmful substances with at least 40
known potent carcinogens.
• Polyaromatic hydrocarbons and nicotine-derived nitrosamine ketone (NNK)
are known to cause DNA damage by forming DNA adducts in animal
models.
• Benzo-A-pyrine also appears to induce molecular signaling such as AKT, as
well as inducing mutations in p53 and other tumor suppressor genes.
Exposure to Asbestos
Exposure to Radon
Outdoor air pollution
Multiple Hit Theory
Pathophysiology
Genetic susceptibility
• Amplification of oncogenes and inactivation of tumor suppressor
genes in NSCLC
ras family of oncogenes - H-ras, K-ras, and N-ras
• Exclusively in adenocarcinoma and are found in 30% of such cases.
• These mutations were not identified in adenocarcinomas that
developed in persons who do not smoke.
• The K-ras mutation appears to be an independent prognostic factor.
Mutations in the oncogenes c-myc and c-raf
APOBEC
Tumor suppressor genes retinoblastoma (Rb) and p53.
Lung Malignancy
Adenocarcinoma
• Arise from bronchial mucosal glands
• Most Common
• Non-Smokers
• Peripheral location within the lung,
• In some cases, at the site of pre-existing scars, wounds, or inflammation (Scar Ca)
Squamous cell carcinoma
• SCC accounts for 25-30% of all
• Central parts of the lung - a cavitary lesion in a proximal bronchus
• Characterized histologically by the presence of keratin pearls
• Can be detected with cytologic studies, as it has a tendency to exfoliate.
• It is the type most often associated with hypercalcemia
Large-cell carcinoma
• Large-cell carcinoma accounts for 10-15%
• As a large peripheral mass
• Sheets of highly atypical cells with focal necrosis - with no evidence of keratinization or gland formation.
Prognosis factors
Symptoms due to Primary tumor
• The symptoms produced by the primary tumor depend on its location
(ie- central, peripheral).
• Central tumors are generally squamous cell carcinomas (SCCs) and produce
cough, dyspnea, atelectasis, postobstructive pneumonia, wheezing, and
hemoptysis.
• Most Peripheral tumors are adenocarcinomas or large cell carcinomas and,
in addition to causing cough and dyspnea, can cause symptoms due to
pleural effusion, and severe pain as a result of infiltration of parietal pleura
and the chest wall.
• Because of their peripheral location, adenocarcinomas may not call attention
to themselves until they have produced extra thoracic metastases. For
example, patients may present with clinical signs of bone spread or
intracranial metastatic disease.
Symptoms due to locoregional spread
• Symptoms due to locoregional spread can include superior vena cava
obstruction,
• Paralysis of the recurrent laryngeal nerve, and phrenic nerve palsy, causing
hoarseness and paralysis of the diaphragm;
• Pressure on the sympathetic plexus, causing Horner syndrome;
• Dysphagia resulting from esophageal compression; and pericardial effusion.
• Superior sulcus tumors (Pancoast tumors) can cause compression of the
brachial plexus roots as they exit the neural foramina, resulting in intense,
radiating neuropathic pain in the ipsilateral upper extremity.
Symptoms due to locoregional spread
Pleural invasion / Mets may produce the following signs and symptoms:
• Chest pain (27-49%)
• Dyspnea (37-58%)
• Cough (45-75%)
Neurologic signs and symptoms include the following:
• Arm weakness and paresthesias (brachial plexus impingement)
• Miosis, ptosis, and anhidrosis (cervical sympathetic chain, Horner
syndrome)
• Dyspnea (secondary to phrenic nerve paralysis)
Metastatic cancer may produce the following signs (8-68%):
• Weight loss
• Cachexia
Symptoms due to distant spread
Central nervous system (CNS) signs and symptoms include the following
• Headache
• Altered mental status
• Seizure
• Meningismus
• Ataxia
• Nausea and/or vomiting
Vascular signs include the following:
• Phlebitis
• Thromboembolism (Trousseau syndrome)
Musculoskeletal manifestations include the following
• Bone pain (6-25%)
• Spinal cord impingement
Differential Diagnoses
• Bacterial Pneumonia
• Bronchitis
• Mycoplasmal Pneumonia
• Pleural Effusion
• Pneumothorax
• Tuberculosis (TB)
• Viral Pneumonia
• Pneumomediastinum
• Pneumonia, Empyema, and abscess
• Pneumothorax, tension and
traumatic
• Benign lung tumors
• Carcinoid lung tumors
• Granuloma
• Hamartoma
• Metastatic cancer
• Small Cell Lung Cancer
• Superior Vena Cava Syndrome in
Emergency Medicine
Diagnosis
• After physical examination and
CBC, chest x-ray is often the first
test performed.
• Chest radiographs may show the
following:
• Pulmonary nodule,
• mass, or infiltrate
• Mediastinal widening
• Atelectasis
• Hilar enlargement
• Pleural effusion
Methods of confirming diagnosis
• Bronchoscopy
• Sputum cytology
• Mediastinoscopy
• Thoracentesis
• Thoracoscopy
• Transthoracic needle biopsy (CT-
or fluoroscopy-guided)
Staging workup
• Invasive staging procedures such as mediastinoscopy and mediastinotomy,
Positron emission tomography (PET) scans may be useful in the detection of
involved nodes,
• The presence of which may influence decisions about operability
• A combination of endosonography and surgical staging had a greater sensitivity
for mediastinal nodal metastases than surgical staging alone
Chest Radiography
• Pulmonary nodule,
mass, or infiltrate
• Mediastinal widening
• Atelectasis
• Hilar enlargement
• Pleural effusion
Non–small cell lung cancer. Left upper collapse
is almost always secondary to endobronchial
bronchogenic carcinoma.
Non–small cell lung cancer. Complete left lung
collapse secondary to bronchogenic carcinoma
of left mainstem bronchus.
Chest Radiography
Chest Radiography
Non–small cell lung cancer. A cavitating
right lower lobe squamous cell
carcinoma.
Non–small cell lung cancer. Right lower lobe
opacity. Not well circumscribed and was found
to be a squamous cell carcinoma
Computed Tomography
• Usually allow a presumptive differentiation between NSCLC and small
cell lung cancer (SCLC).
• Massive lymphadenopathy and direct mediastinal invasion are
commonly associated with small cell carcinoma.
• A mass in or adjacent to the hilum is a particular characteristic of SCLC
Computed Tomography
Lung cancer, small cell. Contrast-enhanced
CT scan of the chest shows a large left lung
and a hilar mass, with invasion of the left
pulmonary artery.
Non–small cell lung cancer. CT scan shows cavitation
and air-fluid level
Magnetic Resonance Imaging
• MRI Spine - Spinal cord compression
• MRI Brain - CNS metastasis
• MRI Chest - Superior sulcus and Brachial plexus tumors
Bone Scintigraphy
• Lung cancer, small cell. Whole-body
nuclear medicine bone scanning with
anterior and posterior images reveal
multiple abnormal areas of increased
radiotracer activity in the pelvis, spine,
ribs, and left scapula.
• These findings are consistent with
bony metastatic disease.
• The bones are commonly affected in
patients with small-cell lung cancer.
Positron Emission Tomography
• Lung cancer, small cell. Coronal
positron emission tomogram shows
abnormal areas of increased metabolic
activity in the left hilar and left adrenal
regions consistent with a hilar tumor
with left adrenal metastasis
Sputum Cytology
• Centrally located endobronchial tumors may exfoliate malignant cells
into sputum ( Esp Squamous Cell Ca)
• False-positive rate for sputum cytology is 1%,
• False-negative rate is as high as 40%.
• Detects 71% of central tumors but less than 50% of peripheral
tumors
• Does not provide reliable distinction between different histologic
subtypes
Bronchoscopy
• Direct visualization of the tumor
• Determination of the extent of airway obstruction
• Collection of diagnostic material under direct visualization
• Direct biopsy of the visualized tumor
• Bronchial brushings and washing, and transbronchial biopsies
Biopsy
• Transthoracic needle biopsy, guided by CT or fluoroscopy
• false-negative rate is high at 26%
• Diagnostic material can also be obtained from other abnormal sites (eg,
enlarged palpable lymph nodes, liver, pleural or pericardial effusions,
accessible bone lesions).
Needle Thoracentesis
• Both diagnostic and therapeutic in patients presenting with respiratory
distress
• sensitivity of only 80% with a specificity greater than 90%.
Thoracoscopy
• Thoracoscopy is usually reserved for tumors that remain undiagnosed
after bronchoscopy or CT-guided biopsy.
• Thoracoscopy is also an important tool in the management of malignant
pleural effusions.
• VATS - used to sample small peripheral tumors (≤2 cm), pleural tumors,
or pleural effusions for diagnostic or staging purposes
Radial Ebus for Peripheral Lung Nodules
Molecular Testing
• Evaluation of patients with metastatic non–small cell lung cancer
(NSCLC).
• EGFR & ALK mutation
• EGFR inhibitors - erlotinib, gefitinib
• ALK inhibitors - crizotinib, ceritinib
• Cobas EGFR Mutation Test - EURTAC study
• EGFR mutations (exon 19 deletions or exon 21 [L858R] substitution
mutations
• Ras mutation is a marker for aggressive disease and poor prognosis
• Regimens to target this mutation - selumetinib plus docetaxel
WHO classification of epithelial lung tumors
Preinvasive lesions
• Squamous dysplasia/carcinoma in situ
• Atypical adenomatous hyperplasia
• Diffuse idiopathic pulmonary neuroendocrine
hyperplasia
Invasive malignant lesions
• Squamous cell carcinoma – Variants, papillary, clear
cell, small cell, basaloid
• Small cell carcinoma – Variant, combined small cell
carcinoma
• Adenocarcinoma
• Acinar,
• papillary,
• bronchoalveolar,
• Non mucinous (Clara cell/type II pneumocyte)
type,
• mixed mucinous and nonmucinous (Clara
cell/type II pneumocyte and goblet cell) type
or intermediate cell type,
• solid adenocarcinoma with mucin formation,
• adenocarcinoma with mixed subtypes,
variants,
• well-differentiated fetal adenocarcinoma,
• mucinous (colloid) adenocarcinoma,
• mucinous cystadenocarcinoma,
• signet-ring adenocarcinoma,
• clear cell adenocarcinoma
WHO classification of epithelial lung tumors
• Large cell carcinoma – Variants,
• large cell neuroendocrine carcinoma,
• combined large cell neuroendocrine carcinoma,
• basaloid carcinoma,
• lymphoepitheliomalike carcinoma, clear cell
carcinoma,
• large cell carcinoma with rhabdoid phenotype
• Adenosquamous carcinoma
• Carcinoma with sarcomatoid,
• pleomorphic, or sarcomatous elements –
• Carcinoma with spindle or giant cells,
• pleomorphic carcinoma,
• spindle cell carcinoma,
• giant cell carcinoma, carcinosarcoma,
• pulmonary blastoma
• Carcinoid tumors –
• Typical carcinoid,
• atypical carcinoid
• Carcinoma of salivary gland type –
• Mucoepidermoid carcinoma,
• adenoid cystic carcinoma,
• others
• Unclassified
Staging
• Chest CT scan is the standard for staging lung cancer
• T describes the size of the primary tumor
• N describes the spread of cancer to regional lymph nodes
• M indicates whether the cancer has metastasized
Staging - Primary tumor (T)
T classification T components on CT
Tis (AIS) Pure GGN ≤ 3 cm
T1 T1mi ≤ 0.5 cm solid part within part-solid tumor total size ≤3 cm
T1a 0.6–1.0 cm solid part within part-solid tumor total size ≤3 cm
Pure GGN >3 cm
≤ 1 cm solid tumor
T1b 1.1–2.0 cm solid part within part-solid tumor total size ≤3 cm
>1–2 cm solid tumor
T1c 2.1–3 cm solid part within part-solid tumor total size ≤3 cm
>2–3 cm solid tumor
T2 T2a 3.1–4 cm Involves main bronchus without involvement of carina
T2b 4.1–5 cm Total/partial atelectasis
Total/partial pneumonitis
Involves hilar fat
Involves visceral pleura (PL1 or PL2)
T3 5.1–7 cm Separate tumor nodules in the same lobe as the primary
Involves parietal pleura (PL3)
Parietal pericardium
Chest wall
Phrenic nerve
T4 >7 cm Involves diaphragm
Mediastinal fat or other mediastinal structures (trachea, great vessels, heart, recurrent laryngeal nerve,
esophagus)
Carina
Vertebral body
Visceral pericardium
Separate tumor nodules in the same lung but different lobes as the primary
Staging - Lymph node (N)
N classification N component on CT
N0 No lymph node metastasis
N1 Ipsilateral peripheral, intrapulmonary or hilar
nodes metastasis
N2 Ipsilateral mediastinal (upper, aortico-
pulmonary, lower), subcarinal nodes metastasis
N3 Ipsilateral or contralateral
supraclavicular/scalene lymph node or
contralateral mediastinal, hilar/interlobar, or
peripheral nodes metastasis
Staging - Metastatic (M)
M classification M component on CT
M0 No distal metastasis
M1 M1a Intrathoracic metastasis
Pleural effusion
Pericardial effusion
Contralateral lung nodules/pleural nodules
M1b Single extrathoracic metastasis in a single organ
M1c Multiple extrathoracic metastasis
Stage
grouping
of
the
TNM
system
N0 N1 N2 N3
M0 Tis 0
T1mi IA1
T1a IA1 IIB IIIA IIIB
T1b IA2 IIB IIIA IIIB
T1c IA3 IIB IIIA IIIB
T2a IB IIB IIIA IIIB
T2b IIA IIB IIIA IIIB
T3 IIB IIIA IIIB IIIC
T4 IIIA IIIA IIIB IIIC
M1a Tx IVA IVA IVA IVA
M1b Tx IVA IVA IVA IVA
M1c Tx IVB IVB IVB IVB
Treatment
• Approach Considerations
• Surgery is the treatment of choice for patients with non–small cell lung cancer
(NSCLC) stages I through IIIA.
• In addition, patients with resected lung cancer have a high risk of relapse and so
are treated with adjuvant chemotherapy.
• Patients with stage IIIB and IV NSCLC are usually offered chemotherapy
• Molecular-targeted therapy plays an increasingly important role in the treatment
of advanced NSCLC.
• Radiation is a reasonable option for treatment in patients who are not candidates
for surgery.
• The role of adjuvant radiation therapy after resection of the primary tumor
remains controversial.
• Palliative care concurrently with standard oncologic care at the initial diagnosis of
advanced NSCLC
Emergency Treatment
• In cases of upper airway obstruction, admit the patient to the intensive
care unit (ICU), prepare for intubation and/or cricothyrotomy, and obtain
otolaryngologic and/or surgical consultation for fiberoptic laryngoscopy
or intraoperative tracheostomy.
• If hemoptysis is noted, administer supplemental oxygen and perform
suctioning. If a threat of imminent demise exists, consider placing a
double-lumen endotracheal tube. Position the patient with the bleeding
hemithorax in a dependent position.
Surgical Treatment
• For all patients with stage I and II NSCLC—that is, those patients with no
evidence of mediastinal disease or invasion of local organs
• Lobectomy is the procedure of choice
• Role of surgery for stage III disease is controversial
• Patients with completely resectable primary tumors (ie, T4 N0) have a
much better prognosis than those with spread to ipsilateral mediastinal
or subcarinal lymph nodes (ie, N2), signifying that spread beyond the
primary tumor is associated with a poor prognosis.
• Patients with stage IIIB or IV tumors are almost never surgical
candidates.
Preoperative evaluation
• Careful assessment of resectability,
• Cardiopulmonary reserve, and perioperative risk.
• High-resolution computed tomography (CT) and positron emission
tomography (PET) scanning are helpful for preoperative planning in early-
stage lung cancer.
• Most patients with a preoperative forced expiratory volume in one second
(FEV1) of greater than 2.5 L are able to tolerate pneumonectomy.
• With an FEV1 of 1.1-2.4 L, a lobectomy is possible.
• Patients with an FEV1 of less than 1 L are not considered candidates for
surgery.
• These factors are further modified by the presence of cardiac disease or other
comorbid conditions.
Postoperative evaluation and complications
• Residual pulmonary function after surgical resection is estimated using
pulmonary function tests and radionuclide lung scans
• The perioperative mortality rate is 6% for pneumonectomy, 3% for
lobectomy, and 1% for segmentectomy. These rates reflect
improvements in anesthesia and surgical techniques.
Radiation Therapy
• In the treatment of stage I and stage II NSCLC, radiation therapy alone is considered only when surgical
resection is not possible because of limited pulmonary reserve or the presence of comorbidities.
• Radiation therapy alone as local therapy, in patients who are not surgical candidates, has been associated
with 5-year cancer specific survival rates of 13-39% in early-stage NSCLC (ie, T1 and T2 disease)
• Survival appears to be enhanced by the use of hyperfractionation schedules, such as continuous
hyperfractionated accelerated radiotherapy (CHART) at 1.5 Gy 3 times a day for 12 days, as opposed to
conventional radiation therapy at 60 Gy in 30 daily fractions. Overall survival at 4 years was 18% vs 12%.
• Independent prognosticators of outcome for hyper fractionated radiation therapy
• female sex, lower Karnofsky performance score (KPS),
• less pronounced weight loss, squamous histology, lower stage,
• shorter inter fraction interval,
• Age did not influence overall survival or progression-free survival.
Radiation Therapy
• Beta blockers affect the metastatic tumors rather than the primary
tumor.
• Complex radiotherapy
• Intermediate complexity radiotherapy
• Stereotactic body radiotherapy (SBRT)
• Radiofrequency ablation (RFA)
• Adjuvant radiation therapy
• Radiation therapy reduces local failures in completely resected
(stages II and IIIA) NSCLC but has not been shown to improve overall
survival rates.
Systemic Chemotherapy
• Approx 80% of all patients with lung cancer are considered for chemotherapy at
some point during the course of their illness
• At present, chemotherapy alone has no role in potentially curative therapy for
NSCLC
• Chemotherapy may be considered as part of multimodality therapy for locally
advanced NSCLC and is used alone in the palliative treatment of stage IIIB NSCLC
(owing to malig plef ) and stage IV NSCLC.
• In advanced NSCLC, patients with good performance status ≥70% on the Karnofsky
scale; and less than 10% body weight loss are good candidates for chemotherapy.
• NSCLC is only moderately sensitive to chemotherapy, with single-agent response
rates in the range of 15% or better.
• Newer agents (eg, gemcitabine, pemetrexed, docetaxel, vinorelbine) have shown
promising single-agent activity, with response rates from 20-25%.
Platinum-based regimens
• In younger patients, with a good performance status or in the adjuvant setting, cisplatin is
preferred, but in older patients or those with significant comorbidities, carboplatin may be
substituted. (ASCO)
• In October 2012, the FDA approved protein-bound paclitaxel (Abraxane) for locally advanced or
metastatic NSCLC, as first-line treatment in combination with carboplatin, in patients who are not
candidates for curative surgery or radiation therapy
• Cisplatin-gemcitabine did appear to have an increased progression free survival, compared with
the standard treatment arm of cisplatin-paclitaxel (4.2 mo vs 3.4 mo), with increased renal toxicity
Histologic factors in chemotherapy responsiveness
• Cisplatin-pemetrexed is now the preferred combination for adenocarcinoma
• squamous cell histology did better with the cisplatin – gemcitabine
Genetic factors in resistance to platinum compounds
• Excision repair cross-complementation group 1 (ERCC1) - high levels in tumor tissue have
been associated with resistance to platinum. ( Significant in Male in comparision to Female )
• Increased expression of ribonucleotide reductase subunit 1 (RRM1) has been associated with
decreased response to gemcitabine and platinum
Second-line chemotherapy
• Docetaxel and pemetrexed have been approved by the FDA in clinical setting of
patients with good responses to first-line chemotherapy, good performance status,
and a long disease-free period between initial chemotherapy and relapse
• Dec 2014, the FDA approved ramucirumab (Cyramza) in combination with
docetaxel for metastatic NSCLC with disease progression on or after platinum-
based chemotherapy
• Chemotherapy can give rise to various adverse effects, as follows:
• Febrile neutropenia or bleeding from bone marrow suppression
• Hyponatremia or hypomagnesemia from cisplatin nephrotoxicity
• Renal failure or ototoxicity from cisplatin
• Peripheral neuropathy from cisplatin, paclitaxel, and vinorelbine
Combined Chemoradiation Therapy
• Standard of care in the management of good-risk (ie, Karnofsky performance score
of 70-100, minimal weight loss) patients with locally advanced unresectable (stage
IIIA) non–small cell lung cancer (NSCLC) is combined-modality therapy consisting
of platinum-based chemotherapy in conjunction with radiation therapy.
• Chemotherapy regimens that have been studied in combination with radiation
therapy include cisplatin/vinblastine and cisplatin/etoposide
• In elderly patients or those with comorbidities and contraindications to cisplatin,
weekly carboplatin/paclitaxel may be used
• Consolidation therapy with the anti-programmed death ligand–1 antibody
durvalumab following concomitant chemoradiation has shown significant benefit
Molecular-Targeted Therapy
• Molecular targeted therapies are revolutionized therapeutics which
interfere with specific molecules to block cancer growth,
progression, and metastasis.
• Many molecular targeted therapies approved by the Food and
Drug Administration (FDA), have demonstrated remarkable clinical
success in the treatment of a myriad of cancer types including
breast, leukemia, colorectal, lung, and ovarian cancers.
Molecular-Targeted Therapy
• All patients with NSCLC should have their tumor tissue tested for mutations, such as in
the genes that code for endothelial growth factor receptor (EGFR), KRAS, anaplastic
lymphoma kinase (ALK), ROS1, and programmed death ligand–1 (PDL1).
• EGFR testing that identifies sensitivity to EGFR-directed tyrosine kinase inhibitors (TKIs)
include exon 19 deletions or the L858R point mutation. The T790M and exon 20 insertion
have been associated with low response or acquired resistance to TKIs.
• In patients with adenocarcinomas that have EGFR mutations consistent with TKI
sensitivity, options for single-agent targeted therapy without chemotherapy include the
following
• Osimertinib (Tagrisso)
• Erlotinib (Tarceva)
• Afatinib (Gilotrif)
• Gefitinib (Iressa)
• Dacomitinib (Vizimpro)
Case Scenario 3
• A 70-year-old male came with complaints of right sided chest pain for
the past 2 months , dyspnea for past 1 month ,swelling of face and
upper limbs for the past 15 days.
• chronic smoker – 50 pack years, chronic alcoholic.
• Driver by occupation.
• On examination - facial
edema +,congested eyes+,
b/l upper limbs swelling +,
dilated veins seen over the
chest wall and around the
scapular areas.
• Rs- decreased breath sound
in right lower hemithorax,
no added sounds.
• Should we need to start treatment for this patient for SVC obstruction
without HPE? or wait for HPE?
Small cell Lung cancer (SCLC)
• Small cell lung cancer (SCLC), previously known
as oat cell carcinoma
• Exhibits aggressive behavior,
• rapid growth,
• early spread to distant sites,
• exquisite sensitivity to chemotherapy and
radiation, and
• association with distinct paraneoplastic
syndromes,
• hypercalcemia
• Eaton-lambert syndrome
• syndrome of inappropriate antidiuretic
hormone (SIADH) secretion
Pathophysiology
• Small cell lung carcinoma (SCLC) arises in peribronchial locations and
infiltrates the bronchial submucosa.
• Widespread metastases occur early in the course of the disease, with
common spread to the mediastinal lymph nodes, liver, bones, adrenal
glands, and brain
Prognosis
• Extensive-stage SCLC Vs Limited-stage disease
• Indicators of poor prognosis include the following:
• Relapsed disease
• Weight loss of greater than 10% of baseline body weight
• Poor performance status
• Hyponatremia
• Elevated LDH level
• Male sex
• Elevated Alkaline Phosphatase level
Complications
• Hypercalcemia could initially be asymptomatic but in late stages could
lead to weakness, fatigue, and sleepiness, and in extreme cases to
severe constipation and lethargy.
• Brain metastasis is often asymptomatic but could manifest as a unilateral
eye abnormality, focal neurologic deficit, or at times with a new-onset
headache that wakes the patient up.
• Seizures
Differential Diagnoses
• Atypical Carcinoid Lung Tumour
• Large Cell Neuroendocrine Carcinoma
• Lung Adenoma
• Lung Hamartoma
• Mediastinal Lymphoma
• Non-Small Cell Lung Cancer (NSCLC)
Histology
• Thought to develop from neuroendocrine Kulchitsky cells and are composed of
sheets of small, round to spindled cells with dark nuclei, scant cytoplasm, and
fine, granular (“salt and pepper”) nuclear chromatin with indistinct nucleoli.
• Very high rates of cell division are observed, and
necrosis, sometimes extensive, may be seen
• Neurosecretory granules can be identified with the aid
of electron microscopy.
• The neuroendocrine nature of the neoplasm is
suggested by its frequent association with neurologic
and endocrine paraneoplastic syndromes
• Immunohistochemical stains for chromogranin, neuron-
specific enolase, CD56, and synaptophysin are usually
positive
Carcinoids SCLC LCNEC
Synaptophysin,
Chromogranin A,
CD56/NCAM, TTF1
(50% are positive +
staining weak and
focal), Estrogen
receptor (50% of
carcinoids) *
Synaptophysin,
Chromogranin A
(only weak), CD56
(most are positive),
cytokeratins
(AE1/AE3 or CAM
5.2), 34ßE12, TTF-
1 (90%)
Synaptophysin,
Chromogranin A
(coexpressed in
70%), TTF-1 (50%)
Common immunohistochemical markers
Staging Overview
• American Cancer Society (ACS) uses 2 types of staging—clinical and
pathologic—for SCLC
• Clinical staging involves physical examination, biopsy examinations, and
imaging scans;
• Majority of patients are staged with clinical staging, and this type of
staging is usually used to describe SCLC tumor extent.
• Pathologic staging is generally more accurate, as it includes clinical
staging and adds postsurgical findings
• Surgical findings may give the cancer a more advanced pathologic stage
Staging systems
VALSG 2-stage system (Veterans’ Administration Lung Study Group)
• Limited-stage and Extensive-stage disease
• Disease confined to one hemithorax, with or without involvement of the
mediastinal, contralateral hilar or ipsilateral supraclavicular, or scalene
lymph nodes are considered to have limited-stage disease, whereas
those with disease involvement at any other location are considered to
have extensive-stage disease
• The key factor in defining limited-stage disease is the ability to
encompass all of the disease within 1 tolerably safe radiation therapy
port
Staging systems
TNM system
• All solid tumors, including lung carcinomas, are staged using the tumor,
node, metastasis (TNM) system, because it provides important prognostic
information and is used to design management plans
• TNM system fails to provide important prognostic information in patients
with SCLC
• AJCC – TNM
• Limited-stage SCLC is defined as any T, any N, M0; the exception is T3-4,
owing to multiple lung nodules that extend beyond a single radiation field
• Extensive-stage disease as any T, any N, M1a/b, and T3-4, due to
involvement of multiple lung nodules
Treatment
Approach Considerations
• Clinical stage Ia (T1N0) after standard staging evaluation may be
considered for surgical resection, but combined treatment with
chemotherapy and radiation therapy is the standard of care. Radiation
therapy is often added at the second cycle of chemotherapy
• Patients with a good performance status and non bulky disease, intensive
radiation therapy early in the course of treatment is indicated
• Elderly patients with SCLC who have a good performance status and intact
organ function should receive standard carboplatin-based chemotherapy
• In 2018 - addition of atezolizumab—a humanized monoclonal anti–
programmed death ligand 1 (PD-L1) antibody—to chemotherapy with
carboplatin and etoposide (IMpower133 study )
Limited-Stage SCLC - Standard Management
• Combination chemotherapy and concurrent thoracic radiotherapy.
• ACCP recommends four cycles of a platinum agent and etoposide
• NCCN guidelines recommend a maximum of four to six cycles of cisplatin
and etoposide
• Radiotherapy - should begin as early as possible, preferably within 30 days
of the start of chemotherapy
• ACCP recommends concurrent accelerated hyper fractionated radiotherapy
(twice-daily treatment) with platinum-based chemotherapy
• NCCN guidelines recommend PCI (Prophylactic Cranial Irradiation) in patients
with limited stage disease who have achieved a complete remission or in
those with stage I disease who have undergone resection
Extensive-Stage SCLC - Standard Management
• Patients with extensive-stage small cell lung cancer (SCLC) are treated
with combination chemotherapy alone. carboplatin or cisplatin plus
etoposide has been the standard of care
• Combination of chemotherapy with atezolizumab—a humanized
monoclonal anti–programmed death ligand 1 (PD-L1) antibody—may
represent a new standard of care.
• Radiotherapy is used only to palliate symptoms, if required in extensive-
stage SCLC
• Gamma knife stereotactic radiosurgery
• a salvage option for patients with brain metastases for whom
previous whole-brain irradiation has failed
Management of Relapsed SCLC
• Patients with relapsed SCLC have an extremely poor prognosis. Individuals whose disease
does not respond to or that progresses on initial treatment or those whose SCLC relapses
within 6 months of completion of therapy have little chance of responding to additional
chemotherapy
• Administration of cisplatin and etoposide (PE) after vincristine (CAV) failure produces
better response rates than does CAV given after PE
Immune checkpoint modulation
• In August 2018, the FDA granted accelerated approval of nivolumab for metastatic SCLC in
patients with progression after platinum-based chemotherapy and at least 1 other line of
therapy
• FDA also granted accelerated approval for pembrolizumab in patients with metastatic SCLC
with disease progression on or after platinum-based chemotherapy and at least 1 other
prior line of therapy
Management of Brain Metastases and Spinal
Cord Compression
• Management of symptomatic brain metastases includes high-dose corticosteroids
and immediate whole brain radiation therapy.
• In asymptomatic brain metastases, systemic chemotherapy may be initiated, with
plans for close surveillance of the central nervous system (CNS) metastases and
initiation of brain radiation after completion of systemic treatment.
Spinal cord compression
• An oncologic emergency
• IV corticosteroids even before being sent for MRI
• Radiation therapy and/or neurosurgical decompression, undertaken without
delay.
Management of Complications
• Tumor lysis syndrome
• Laboratory features of tumor lysis syndrome are hyperuricemia,
hyperphosphatemia, hypocalcemia, and hyperkalemia
• Patients should be well hydrated and, preferably, premedicated with allopurinol
• Management of established tumor lysis syndrome is urinary alkalinization,
correction of electrolyte abnormalities, and dialysis
• Electrolyte abnormalities
• Most common abnormality is hyponatremia, which, if severe, may cause
neurologic symptoms and signs, including seizures, coma, and death
• Other causes of hyponatremia (ie, volume depletion, abnormal renal function)
must be excluded.
• Fluid restriction and pharmacologic therapy in the form of demeclocycline
SVC Syndrome
Definition:
The clinical signs and symptoms and radiographic manifestations
resulting from obstruction of blood flow to the heart from the head and
neck regions and upper extremities as a consequence of compression of
the superior vena cava, either from direct invasion by the primary tumor
into the mediastinum or from lymphatic spread with enlarged right
paratracheal lymph nodes
Signs of SVCS
• Feeling of fullness in the head
• Dyspnea
• Cough
• Dilated neck veins Prominent venous pattern on the face and the chest
• Upper extremity and facial edema
• Papilledema
• Facial cyanosis
• Plethora
• Conjunctival edema
Grades of SVCS
• Grade 0 - SVC narrowing without clinical evidence of SVCS
• Grade I - Moderate SVC narrowing without collateral
• Grade II - Severe SVC narrowing with the azygos vein serving as partial
collateral
• Grade III - SVC obstruction above the azygos arch
• Grade IV - SVC obstruction at or below the level of the azygos arch
Collaterals in SVC Syndrome
(a)Azygos-hemiazygos,
(b)Paravertebral,
(c)InternalMammary,
(d)Anteriorjugular,
(e)Thymic,
(f)Lateralthoracic,
(g)EsophagealVenous
System
(h)Thyroidal,
(i)Pericardiophrenic
Azygos and Hemiazygos
Venous System
Paravertebral Venous
System
Therapeutic Considerations
• Goals of Therapy
• Palliation of symptoms in the incurable situation
• Aggressive management in the potentially curable situation.
• Histologic diagnosis and staging are vital to determine whether
palliation or cure is the goal
• When the etiology is nonmalignant, relief of the SVC obstruction is the
goal
• Minimizing treatment-related morbidity is an important secondary goal.
Symptomatic Relief
• Head end elevation
• Emergency radiotherapy
• Glucocorticoids
• Diuretics
Head end elevation
• Decreases hydrostatic pressure in upper body
• Reduces head & neck edema
• Not clinically proven
Emergency radiotherapy
• Not needed for most of patients
• Obstruction is a chronic process. Emergency RT provides no benefit as
long as the patients is clinically stable
• May obscure diagnosis if biopsy is planned later
Glucocorticoids
• Laryngeal edema
• Cerebral edema
• Steroid responsive tumours - lymphoma / thymomas
Diuretics
• Commonly used
• Not clinically proven
Treatment of underlying cause
• Stenting
• Radiotherapy
• Chemotherapy
• Surgery
Stenting
• Rapid & sustained relief
• Success rate 95-100 %
• Under local anaesthesia
• Per cutaneous procedure
• Self expanding stents are preferred
• May require more than one stents
• Total occlusion is not a contraindication
Stenting
Stenting
Surgical Reconstruction
• Venous autografts produce the best results but are not always feasible
• Pedicle created from the right atrium or the pericardium,
• Aortic homografts
• Synthetic grafts
• Do not have an antigenicity problem
• But are susceptible to intraluminal thrombosis
• Patency rate of polytetrafluoroethylene grafts - 62%
SURVIVAL
• Mean survival for patients with malignancy ranged from 12.0 to 40.2
weeks
• Median survival of all patients was 22 weeks
• 5-year survival rate decreasing from 41% for lymphoma to 5% for small
cell lung cancer to 2% for NSCLC
Paraneoplastic Syndromes
Definition
• A paraneoplastic syndrome is a disease or symptom that is the
consequence of the presence of cancer in the body, but is not due to the
local presence of cancer cells.
• Two of the most common are humoral hypercalcemia of malignancy
(HHM) in squamous cell carcinoma and the syndrome of inappropriate
antidiuretic hormone secretion (SIADH) in small cell lung cancer.
• These phenomena are mediated by humoral factors (by hormones or
cytokines)excreted by tumor cells or by an immune response against the
tumor.
Importance
• Paraneoplastic syndromes occur in about 10% - 15% of malignant disease.
• They may represent the earliest manifestation of an occult neoplasm.
• They may mimic metastatic disease and therefore confound treatment.
• They may serve as a TUMOR MARKER in previously treated patients to
detect recurrence.
• In patients undergoing ADJUVANT THERAPY to guide further treatment.
CLASSIFICATION OF PNS
Paraneoplastic endocrine syndromes associated
with lung cancer
Humoral hypercalcemia of malignancy
Syndrome of inappropriate antidiuretic hormone production
Cushing’s syndrome
Hypoglycemia
Acromegaly
Carcinoid syndrome
Gynecomastia
Hyperthyroidism
Criteria for the diagnosis of paraneoplastic
endocrine syndromes
• Abnormal endocrine function without physiologic feedback regulation
• The absence of metastasis in the respective endocrine gland
• Deterioration with increasing tumor burden
• Improvement in endocrine function with the treatment of the tumor
• Evidence of the presence of hormones in the tumor or hormone
synthesis by the tumor
Causes of hypercalcemia associated with lung
cancer
Humoral hypercalcemia of malignancy
(1) Parathyroid hormone-related protein
(2) Parathyroid hormone
(3) 1,25-dihydroxyvitamin D
(4) Granulocyte colony-stimulating factor
Osteolytic activity at the sites of skeletal metastases
Criteria for the diagnosis of Syndrome of
inappropriate antidiuretic hormone
Hyponatremia (serum sodium < 134 mEq/L)
Hypoosmolality (plasma osmolality < 275 mOsm/kg)
Inappropriately high urine osmolality ( > 500 mOsm/kg)
Inappropriately high urinary sodium concentration ( > 20 mEq/L)
Absence of hypothyroidism
Absence of adrenal insufficiency
Absence of volume depletion
Treatment
• Treatment of the underlying tumor.
• Discontinue medications that contribute to hypercalcemia (eg, calcium
supplements, vitamin D, thiazide diuretics, calcium-containing antacids,
and lithium .
• Fluid repletion with normal saline, which increases the glomerular
filtration rate and inhibits renal calcium reabsorption.
• IV bisphosphonates, Calcitonin
• Mithramycin blocks bone resorption by inhibiting osteoclast RNA
synthesis, Gallium nitrate-inhibition of osteoclastic bone resorption
CONCLUSION
• A paraneoplastic syndromes affect the presentation, clinical course, and
treatment of cancer.
• As a result of recent diagnostic and therapeutic advances, many
paraneoplastic syndromes are currently well defined, have a clear
pathogenesis, and have effective treatment options.
• The ability to recognize and treat paraneoplastic syndromes may have a
substantial effect on clinical outcomes, ranging from earlier cancer
diagnosis, to improved quality of life, to increased delivery of tumor-
directed therapy.
Take Home points
• High degree of suspicion important to diagnose Lung cancer at earlier
stage
• Screening for Lung cancer
• Least invasive method should be employed to diagnose and stage the
disease
• At least appropriate treatment to alleviate the symptoms need to be
taken care if no definite management possible.
THANK YOU

More Related Content

Similar to Thoracic Malignancy.pdf

management of lung mets
management of lung metsmanagement of lung mets
management of lung metssuhas k r
 
Locally advanced lung ca
Locally advanced lung caLocally advanced lung ca
Locally advanced lung caamanuelasefa1
 
NSCLC housestaff
NSCLC housestaffNSCLC housestaff
NSCLC housestaffderosaMSKCC
 
Ca Papillary(Thyroid Gland)
Ca Papillary(Thyroid Gland)Ca Papillary(Thyroid Gland)
Ca Papillary(Thyroid Gland)DRASIMSHAHZAD1
 
cancer papilar prezentare.pptx
cancer papilar prezentare.pptxcancer papilar prezentare.pptx
cancer papilar prezentare.pptxssuser311078
 
Lung Cancer Screening
Lung Cancer ScreeningLung Cancer Screening
Lung Cancer ScreeningGamal Agmy
 
Focussed therapy and imaging in prostate cancer
Focussed therapy and imaging in prostate cancerFocussed therapy and imaging in prostate cancer
Focussed therapy and imaging in prostate cancerPrateek Laddha
 
REVIEW OF METHODS FOR DIAGNOSIS OF LUNG CANCER
REVIEW OF METHODS FOR DIAGNOSIS OF LUNG CANCERREVIEW OF METHODS FOR DIAGNOSIS OF LUNG CANCER
REVIEW OF METHODS FOR DIAGNOSIS OF LUNG CANCERswankyshahir
 
lungcancer-171013101354.pptx
lungcancer-171013101354.pptxlungcancer-171013101354.pptx
lungcancer-171013101354.pptxNehaPandey199
 
Carcinoma anal canal - Dr Naina kumar agarwal MCh surgical oncology resident
Carcinoma anal canal - Dr Naina kumar agarwal MCh surgical oncology residentCarcinoma anal canal - Dr Naina kumar agarwal MCh surgical oncology resident
Carcinoma anal canal - Dr Naina kumar agarwal MCh surgical oncology residentDr. Naina Kumar Agarwal
 
Low Dose CT Screening for Early Diagnosis of Lung Cancer
Low Dose CT Screening for Early Diagnosis of Lung CancerLow Dose CT Screening for Early Diagnosis of Lung Cancer
Low Dose CT Screening for Early Diagnosis of Lung CancerKue Lee
 
Lang cancer.pdf
Lang cancer.pdfLang cancer.pdf
Lang cancer.pdfssuser3a0aee
 
Lung Cancer: Diagnosis, Staging, and Treatment
Lung Cancer: Diagnosis, Staging, and TreatmentLung Cancer: Diagnosis, Staging, and Treatment
Lung Cancer: Diagnosis, Staging, and TreatmentDene W. Daugherty
 
Solitary pulmonary nodule
Solitary pulmonary noduleSolitary pulmonary nodule
Solitary pulmonary noduleDileep Benji
 
LungCancerSlides.pptx
LungCancerSlides.pptxLungCancerSlides.pptx
LungCancerSlides.pptxHesocaHux
 
Lung cancer
Lung cancerLung cancer
Lung cancerjamal turki
 

Similar to Thoracic Malignancy.pdf (20)

management of lung mets
management of lung metsmanagement of lung mets
management of lung mets
 
Locally advanced lung ca
Locally advanced lung caLocally advanced lung ca
Locally advanced lung ca
 
NSCLC housestaff
NSCLC housestaffNSCLC housestaff
NSCLC housestaff
 
Ca Papillary(Thyroid Gland)
Ca Papillary(Thyroid Gland)Ca Papillary(Thyroid Gland)
Ca Papillary(Thyroid Gland)
 
cancer papilar prezentare.pptx
cancer papilar prezentare.pptxcancer papilar prezentare.pptx
cancer papilar prezentare.pptx
 
Lung tumor
Lung tumorLung tumor
Lung tumor
 
Lung Cancer Screening
Lung Cancer ScreeningLung Cancer Screening
Lung Cancer Screening
 
Focussed therapy and imaging in prostate cancer
Focussed therapy and imaging in prostate cancerFocussed therapy and imaging in prostate cancer
Focussed therapy and imaging in prostate cancer
 
LUNG CANCER
LUNG CANCERLUNG CANCER
LUNG CANCER
 
REVIEW OF METHODS FOR DIAGNOSIS OF LUNG CANCER
REVIEW OF METHODS FOR DIAGNOSIS OF LUNG CANCERREVIEW OF METHODS FOR DIAGNOSIS OF LUNG CANCER
REVIEW OF METHODS FOR DIAGNOSIS OF LUNG CANCER
 
lungcancer-171013101354.pptx
lungcancer-171013101354.pptxlungcancer-171013101354.pptx
lungcancer-171013101354.pptx
 
Carcinoma anal canal - Dr Naina kumar agarwal MCh surgical oncology resident
Carcinoma anal canal - Dr Naina kumar agarwal MCh surgical oncology residentCarcinoma anal canal - Dr Naina kumar agarwal MCh surgical oncology resident
Carcinoma anal canal - Dr Naina kumar agarwal MCh surgical oncology resident
 
Low Dose CT Screening for Early Diagnosis of Lung Cancer
Low Dose CT Screening for Early Diagnosis of Lung CancerLow Dose CT Screening for Early Diagnosis of Lung Cancer
Low Dose CT Screening for Early Diagnosis of Lung Cancer
 
Lang cancer.pdf
Lang cancer.pdfLang cancer.pdf
Lang cancer.pdf
 
Lung Cancer: Diagnosis, Staging, and Treatment
Lung Cancer: Diagnosis, Staging, and TreatmentLung Cancer: Diagnosis, Staging, and Treatment
Lung Cancer: Diagnosis, Staging, and Treatment
 
Solitary pulmonary nodule
Solitary pulmonary noduleSolitary pulmonary nodule
Solitary pulmonary nodule
 
Lung cancer seminar
Lung cancer seminarLung cancer seminar
Lung cancer seminar
 
Non small cell ca
Non small cell caNon small cell ca
Non small cell ca
 
LungCancerSlides.pptx
LungCancerSlides.pptxLungCancerSlides.pptx
LungCancerSlides.pptx
 
Lung cancer
Lung cancerLung cancer
Lung cancer
 

More from drperumal

Mycophenalate.pptx
Mycophenalate.pptxMycophenalate.pptx
Mycophenalate.pptxdrperumal
 
Police Health care and Pandemic _SRP.pptx
Police Health care and Pandemic _SRP.pptxPolice Health care and Pandemic _SRP.pptx
Police Health care and Pandemic _SRP.pptxdrperumal
 
Nebulisation & Medications.pptx
Nebulisation & Medications.pptxNebulisation & Medications.pptx
Nebulisation & Medications.pptxdrperumal
 
Clinical History & General Examination.pptx
Clinical History & General Examination.pptxClinical History & General Examination.pptx
Clinical History & General Examination.pptxdrperumal
 
Bronchial Asthma 2022.pptx
Bronchial Asthma 2022.pptxBronchial Asthma 2022.pptx
Bronchial Asthma 2022.pptxdrperumal
 
VAP Prevention.pptx
VAP Prevention.pptxVAP Prevention.pptx
VAP Prevention.pptxdrperumal
 

More from drperumal (6)

Mycophenalate.pptx
Mycophenalate.pptxMycophenalate.pptx
Mycophenalate.pptx
 
Police Health care and Pandemic _SRP.pptx
Police Health care and Pandemic _SRP.pptxPolice Health care and Pandemic _SRP.pptx
Police Health care and Pandemic _SRP.pptx
 
Nebulisation & Medications.pptx
Nebulisation & Medications.pptxNebulisation & Medications.pptx
Nebulisation & Medications.pptx
 
Clinical History & General Examination.pptx
Clinical History & General Examination.pptxClinical History & General Examination.pptx
Clinical History & General Examination.pptx
 
Bronchial Asthma 2022.pptx
Bronchial Asthma 2022.pptxBronchial Asthma 2022.pptx
Bronchial Asthma 2022.pptx
 
VAP Prevention.pptx
VAP Prevention.pptxVAP Prevention.pptx
VAP Prevention.pptx
 

Recently uploaded

Dehradun Call Girls Service ❤️🍑 9675010100 👄🫦Independent Escort Service Dehradun
Dehradun Call Girls Service ❤️🍑 9675010100 👄🫦Independent Escort Service DehradunDehradun Call Girls Service ❤️🍑 9675010100 👄🫦Independent Escort Service Dehradun
Dehradun Call Girls Service ❤️🍑 9675010100 👄🫦Independent Escort Service DehradunNiamh verma
 
Russian Call Girls in Hyderabad Ishita 9907093804 Independent Escort Service ...
Russian Call Girls in Hyderabad Ishita 9907093804 Independent Escort Service ...Russian Call Girls in Hyderabad Ishita 9907093804 Independent Escort Service ...
Russian Call Girls in Hyderabad Ishita 9907093804 Independent Escort Service ...delhimodelshub1
 
Call Girls in Hyderabad Lavanya 9907093804 Independent Escort Service Hyderabad
Call Girls in Hyderabad Lavanya 9907093804 Independent Escort Service HyderabadCall Girls in Hyderabad Lavanya 9907093804 Independent Escort Service Hyderabad
Call Girls in Hyderabad Lavanya 9907093804 Independent Escort Service Hyderabaddelhimodelshub1
 
Russian Call Girls in Raipur 9873940964 Book Hot And Sexy Girls
Russian Call Girls in Raipur 9873940964 Book Hot And Sexy GirlsRussian Call Girls in Raipur 9873940964 Book Hot And Sexy Girls
Russian Call Girls in Raipur 9873940964 Book Hot And Sexy Girlsddev2574
 
Dehradun Call Girls Service 7017441440 Real Russian Girls Looking Models
Dehradun Call Girls Service 7017441440 Real Russian Girls Looking ModelsDehradun Call Girls Service 7017441440 Real Russian Girls Looking Models
Dehradun Call Girls Service 7017441440 Real Russian Girls Looking Modelsindiancallgirl4rent
 
Russian Call Girls Hyderabad Indira 9907093804 Independent Escort Service Hyd...
Russian Call Girls Hyderabad Indira 9907093804 Independent Escort Service Hyd...Russian Call Girls Hyderabad Indira 9907093804 Independent Escort Service Hyd...
Russian Call Girls Hyderabad Indira 9907093804 Independent Escort Service Hyd...delhimodelshub1
 
No Advance 9053900678 Chandigarh Call Girls , Indian Call Girls For Full Ni...
No Advance 9053900678 Chandigarh  Call Girls , Indian Call Girls  For Full Ni...No Advance 9053900678 Chandigarh  Call Girls , Indian Call Girls  For Full Ni...
No Advance 9053900678 Chandigarh Call Girls , Indian Call Girls For Full Ni...Vip call girls In Chandigarh
 
Russian Call Girls Hyderabad Saloni 9907093804 Independent Escort Service Hyd...
Russian Call Girls Hyderabad Saloni 9907093804 Independent Escort Service Hyd...Russian Call Girls Hyderabad Saloni 9907093804 Independent Escort Service Hyd...
Russian Call Girls Hyderabad Saloni 9907093804 Independent Escort Service Hyd...delhimodelshub1
 
College Call Girls Hyderabad Sakshi 9907093804 Independent Escort Service Hyd...
College Call Girls Hyderabad Sakshi 9907093804 Independent Escort Service Hyd...College Call Girls Hyderabad Sakshi 9907093804 Independent Escort Service Hyd...
College Call Girls Hyderabad Sakshi 9907093804 Independent Escort Service Hyd...delhimodelshub1
 
Vip sexy Call Girls Service In Sector 137,9999965857 Young Female Escorts Ser...
Vip sexy Call Girls Service In Sector 137,9999965857 Young Female Escorts Ser...Vip sexy Call Girls Service In Sector 137,9999965857 Young Female Escorts Ser...
Vip sexy Call Girls Service In Sector 137,9999965857 Young Female Escorts Ser...Call Girls Noida
 
Call Girls Service Chandigarh Gori WhatsApp ❤9115573837 VIP Call Girls Chandi...
Call Girls Service Chandigarh Gori WhatsApp ❤9115573837 VIP Call Girls Chandi...Call Girls Service Chandigarh Gori WhatsApp ❤9115573837 VIP Call Girls Chandi...
Call Girls Service Chandigarh Gori WhatsApp ❤9115573837 VIP Call Girls Chandi...Niamh verma
 
indian Call Girl Panchkula ❤️🍑 9907093804 Low Rate Call Girls Ludhiana Tulsi
indian Call Girl Panchkula ❤️🍑 9907093804 Low Rate Call Girls Ludhiana Tulsiindian Call Girl Panchkula ❤️🍑 9907093804 Low Rate Call Girls Ludhiana Tulsi
indian Call Girl Panchkula ❤️🍑 9907093804 Low Rate Call Girls Ludhiana TulsiHigh Profile Call Girls Chandigarh Aarushi
 
VIP Call Girl Sector 25 Gurgaon Just Call Me 9899900591
VIP Call Girl Sector 25 Gurgaon Just Call Me 9899900591VIP Call Girl Sector 25 Gurgaon Just Call Me 9899900591
VIP Call Girl Sector 25 Gurgaon Just Call Me 9899900591adityaroy0215
 
Leading transformational change: inner and outer skills
Leading transformational change: inner and outer skillsLeading transformational change: inner and outer skills
Leading transformational change: inner and outer skillsHelenBevan4
 
Gurgaon iffco chowk 🔝 Call Girls Service 🔝 ( 8264348440 ) unlimited hard sex ...
Gurgaon iffco chowk 🔝 Call Girls Service 🔝 ( 8264348440 ) unlimited hard sex ...Gurgaon iffco chowk 🔝 Call Girls Service 🔝 ( 8264348440 ) unlimited hard sex ...
Gurgaon iffco chowk 🔝 Call Girls Service 🔝 ( 8264348440 ) unlimited hard sex ...soniya singh
 
Call Girls Hyderabad Kirti 9907093804 Independent Escort Service Hyderabad
Call Girls Hyderabad Kirti 9907093804 Independent Escort Service HyderabadCall Girls Hyderabad Kirti 9907093804 Independent Escort Service Hyderabad
Call Girls Hyderabad Kirti 9907093804 Independent Escort Service Hyderabaddelhimodelshub1
 
Call Girls Hyderabad Krisha 9907093804 Independent Escort Service Hyderabad
Call Girls Hyderabad Krisha 9907093804 Independent Escort Service HyderabadCall Girls Hyderabad Krisha 9907093804 Independent Escort Service Hyderabad
Call Girls Hyderabad Krisha 9907093804 Independent Escort Service Hyderabaddelhimodelshub1
 
Call Girls Kukatpally 7001305949 all area service COD available Any Time
Call Girls Kukatpally 7001305949 all area service COD available Any TimeCall Girls Kukatpally 7001305949 all area service COD available Any Time
Call Girls Kukatpally 7001305949 all area service COD available Any Timedelhimodelshub1
 

Recently uploaded (20)

Dehradun Call Girls Service ❤️🍑 9675010100 👄🫦Independent Escort Service Dehradun
Dehradun Call Girls Service ❤️🍑 9675010100 👄🫦Independent Escort Service DehradunDehradun Call Girls Service ❤️🍑 9675010100 👄🫦Independent Escort Service Dehradun
Dehradun Call Girls Service ❤️🍑 9675010100 👄🫦Independent Escort Service Dehradun
 
Russian Call Girls in Hyderabad Ishita 9907093804 Independent Escort Service ...
Russian Call Girls in Hyderabad Ishita 9907093804 Independent Escort Service ...Russian Call Girls in Hyderabad Ishita 9907093804 Independent Escort Service ...
Russian Call Girls in Hyderabad Ishita 9907093804 Independent Escort Service ...
 
Call Girls in Hyderabad Lavanya 9907093804 Independent Escort Service Hyderabad
Call Girls in Hyderabad Lavanya 9907093804 Independent Escort Service HyderabadCall Girls in Hyderabad Lavanya 9907093804 Independent Escort Service Hyderabad
Call Girls in Hyderabad Lavanya 9907093804 Independent Escort Service Hyderabad
 
Russian Call Girls in Raipur 9873940964 Book Hot And Sexy Girls
Russian Call Girls in Raipur 9873940964 Book Hot And Sexy GirlsRussian Call Girls in Raipur 9873940964 Book Hot And Sexy Girls
Russian Call Girls in Raipur 9873940964 Book Hot And Sexy Girls
 
Dehradun Call Girls Service 7017441440 Real Russian Girls Looking Models
Dehradun Call Girls Service 7017441440 Real Russian Girls Looking ModelsDehradun Call Girls Service 7017441440 Real Russian Girls Looking Models
Dehradun Call Girls Service 7017441440 Real Russian Girls Looking Models
 
Russian Call Girls Hyderabad Indira 9907093804 Independent Escort Service Hyd...
Russian Call Girls Hyderabad Indira 9907093804 Independent Escort Service Hyd...Russian Call Girls Hyderabad Indira 9907093804 Independent Escort Service Hyd...
Russian Call Girls Hyderabad Indira 9907093804 Independent Escort Service Hyd...
 
No Advance 9053900678 Chandigarh Call Girls , Indian Call Girls For Full Ni...
No Advance 9053900678 Chandigarh  Call Girls , Indian Call Girls  For Full Ni...No Advance 9053900678 Chandigarh  Call Girls , Indian Call Girls  For Full Ni...
No Advance 9053900678 Chandigarh Call Girls , Indian Call Girls For Full Ni...
 
Russian Call Girls Hyderabad Saloni 9907093804 Independent Escort Service Hyd...
Russian Call Girls Hyderabad Saloni 9907093804 Independent Escort Service Hyd...Russian Call Girls Hyderabad Saloni 9907093804 Independent Escort Service Hyd...
Russian Call Girls Hyderabad Saloni 9907093804 Independent Escort Service Hyd...
 
College Call Girls Hyderabad Sakshi 9907093804 Independent Escort Service Hyd...
College Call Girls Hyderabad Sakshi 9907093804 Independent Escort Service Hyd...College Call Girls Hyderabad Sakshi 9907093804 Independent Escort Service Hyd...
College Call Girls Hyderabad Sakshi 9907093804 Independent Escort Service Hyd...
 
Vip sexy Call Girls Service In Sector 137,9999965857 Young Female Escorts Ser...
Vip sexy Call Girls Service In Sector 137,9999965857 Young Female Escorts Ser...Vip sexy Call Girls Service In Sector 137,9999965857 Young Female Escorts Ser...
Vip sexy Call Girls Service In Sector 137,9999965857 Young Female Escorts Ser...
 
Call Girls Service Chandigarh Gori WhatsApp ❤9115573837 VIP Call Girls Chandi...
Call Girls Service Chandigarh Gori WhatsApp ❤9115573837 VIP Call Girls Chandi...Call Girls Service Chandigarh Gori WhatsApp ❤9115573837 VIP Call Girls Chandi...
Call Girls Service Chandigarh Gori WhatsApp ❤9115573837 VIP Call Girls Chandi...
 
indian Call Girl Panchkula ❤️🍑 9907093804 Low Rate Call Girls Ludhiana Tulsi
indian Call Girl Panchkula ❤️🍑 9907093804 Low Rate Call Girls Ludhiana Tulsiindian Call Girl Panchkula ❤️🍑 9907093804 Low Rate Call Girls Ludhiana Tulsi
indian Call Girl Panchkula ❤️🍑 9907093804 Low Rate Call Girls Ludhiana Tulsi
 
VIP Call Girl Sector 25 Gurgaon Just Call Me 9899900591
VIP Call Girl Sector 25 Gurgaon Just Call Me 9899900591VIP Call Girl Sector 25 Gurgaon Just Call Me 9899900591
VIP Call Girl Sector 25 Gurgaon Just Call Me 9899900591
 
Leading transformational change: inner and outer skills
Leading transformational change: inner and outer skillsLeading transformational change: inner and outer skills
Leading transformational change: inner and outer skills
 
VIP Call Girls Lucknow Isha 🔝 9719455033 🔝 🎶 Independent Escort Service Lucknow
VIP Call Girls Lucknow Isha 🔝 9719455033 🔝 🎶 Independent Escort Service LucknowVIP Call Girls Lucknow Isha 🔝 9719455033 🔝 🎶 Independent Escort Service Lucknow
VIP Call Girls Lucknow Isha 🔝 9719455033 🔝 🎶 Independent Escort Service Lucknow
 
Gurgaon iffco chowk 🔝 Call Girls Service 🔝 ( 8264348440 ) unlimited hard sex ...
Gurgaon iffco chowk 🔝 Call Girls Service 🔝 ( 8264348440 ) unlimited hard sex ...Gurgaon iffco chowk 🔝 Call Girls Service 🔝 ( 8264348440 ) unlimited hard sex ...
Gurgaon iffco chowk 🔝 Call Girls Service 🔝 ( 8264348440 ) unlimited hard sex ...
 
Call Girls Hyderabad Kirti 9907093804 Independent Escort Service Hyderabad
Call Girls Hyderabad Kirti 9907093804 Independent Escort Service HyderabadCall Girls Hyderabad Kirti 9907093804 Independent Escort Service Hyderabad
Call Girls Hyderabad Kirti 9907093804 Independent Escort Service Hyderabad
 
Call Girls Hyderabad Krisha 9907093804 Independent Escort Service Hyderabad
Call Girls Hyderabad Krisha 9907093804 Independent Escort Service HyderabadCall Girls Hyderabad Krisha 9907093804 Independent Escort Service Hyderabad
Call Girls Hyderabad Krisha 9907093804 Independent Escort Service Hyderabad
 
Call Girls Kukatpally 7001305949 all area service COD available Any Time
Call Girls Kukatpally 7001305949 all area service COD available Any TimeCall Girls Kukatpally 7001305949 all area service COD available Any Time
Call Girls Kukatpally 7001305949 all area service COD available Any Time
 
#9711199012# African Student Escorts in Delhi 😘 Call Girls Delhi
#9711199012# African Student Escorts in Delhi 😘 Call Girls Delhi#9711199012# African Student Escorts in Delhi 😘 Call Girls Delhi
#9711199012# African Student Escorts in Delhi 😘 Call Girls Delhi
 

Thoracic Malignancy.pdf

  • 1. THORACIC MALIGNANCY • Prof.Dr.A.SundararajaperumalM.D;D.C.H • Professor • Institute Of Thoracic Medicine • Madras Medical College & RGGGH • Chennai - 3
  • 2. Overview Lung Cancer Screening Fleischner’s Guidelines Diagnostic workup of Lung Cancer Staging of Lung Cancer Complications
  • 3. Case Scenario 1 A 60 yrs old, Mr.G comes to our hospital with chronic cough. Chronic smoker, 30 pack years. A farmer by occupation. He is a known case of COPD/CAD/past history of surgery for hydrocele. He is currently on LAMA/LABA inhaler 2 puffs daily, on dual antiplatelets. He is other wise able to carry out all his activities on his own. His son wants to know if his father could develop lung cancer and if there are any screening tests.
  • 4. Question 1 • 1. Screening tests are not very effective in Lung cancer • 2. Screening test with low dose CT scan is an option but this patient is not a suitable candidate. • 3.Screening can be offered to him at 1,2,3.5yrs respectively • 4.Screening can be offered to him at 1,2,2.5 yrs respectively.
  • 5. Screening – Lung Malignancy • Only recommended screening test for lung cancer is Low-dose computed tomography (also called a low-dose CT scan, or LDCT). • Who Should Be Screened?  Have a 20 pack-year or more smoking history, and  Smoke now or have quit within the past 15 years, and  Are between 50 and 80 years old.
  • 6. The National Lung Screening Trial (NLST) • Evaluated annual LDCT vs CXR screening in 53,454 patients aged 55–74 y with a 30-pack year smoking • History (including those who quit within 15 y), showing a 20% reduction in lung cancer and 6.7% reduction in all-cause mortality in the CT arm. • 320 patients would need to be screened to prevent one lung cancer death.
  • 7. NELSON RCT-EUROPE • The NELSON European RCT included 15,822 current/former smokers • Evaluated LDCT screening at increasing screening intervals (1, 2, and 2.5 years), compared with no screening. • Presented data suggests male mortality reduction of 26% at 10 y, female mortality reduction of 39% at 10 y
  • 8. Pulmonary Nodules • Pulmonary nodules are frequently encountered on chest CT screening • Nodule characterization should be performed on thin-slice CT images ≤1.5 mm, since a small solid nodule may appear to have groundglass density on a thick slice due to partial-volume effect.
  • 9. Pulmonary Nodule Pulmonary nodules can be divided into solid lesions and subsolid lesions, which can be further subdivided into part-solid and pure ground glass nodules. Subsolid nodule (SSN) A pulmonary nodule with at least partial groundglass appearance Groundglass Opacification with a higher density than the surrounding tissue, not obscuring underlying bronchovascular structures
  • 10. Subsolid nodules • Most subsolid nodules are transient and the result of infection or hemorrhage. • However, persistent subsolid nodules often represent pathology in the adenocarcinomatous spectrum. • No reliable distinction can be made radiologically, although studies suggest that larger size and a solid component are associated with more invasive behaviour. • Compared to solid lesions, persistent subsolid nodules have a much slower growth rate, but carry a much higher risk of malignancy. • In a study by Henschke et al., part-solid nodules were malignant in 63%, pure groundglass SSNs in 18% and solid nodules only in 7%
  • 12. Solid nodules Solid pulmonary nodules can represent various etiologies • Benign granulomas • Focal scar • Intrapulmonary lymph nodes • Primary malignancies • Metastatic disease.
  • 14. Probability of Malignancy Low ( ˂5% ) Intermediate ( 5-65% ) High ( ˃65% ) • Young • Less smoking • No Prior Cancer • Small nodule size • Regular margin • Non-Upper Lobe • Mixture of Low and High probability features • Older • Heavy Smoking • Prior Cancer • Larger size • Irregular Margin • Upper lobe location • Emphysema • Pulmonary fibrosis ( IPF ) • Family H/O Lung cancer
  • 15. Fleischner guideline for Pulmonary nodule management • Only apply in subjects 35 years or older • Do not apply in immunocompromised subjects or pts with a known or suspected malignancy • Use Thin-slice CT imaging ( LDCT ) • Manual nodule measurements should be based on the avg of long and short axis diameters. Alternatively use nodule volumetry. • Risk - stratification on both patient and nodule charectristics
  • 16. Perifissural nodule • Perifissural nodules are a separate and benign entity. • A typical PFN is attached to a pulmonary fissure, is homogeneous, and solid with smooth margins. The shape is oval, lentiform or triangular • A nodule with these specific characteristics needs no follow-up and is probably an intrapulmonary lymph node. • In a study by de Hoop none of the 919 typical and atypical PFNs were found to be malignant in 5.5 year follow-up. • Typical PFNs can show significant growth rates on serial imaging comparable to malignant nodules. • This is not a sign of malignancy, but merely a result of their lymphatic origin.
  • 17. CT Chest of the patient
  • 18. When Should Screening Stop?  Turns 81 years old, or  Has not smoked in 15 or more years, or  Develops a health problem that makes him or her unwilling or unable to have surgery if lung cancer is found.
  • 19. Biomarker screening • Cancer antigen 125 • Carcinoembryonic antigen • Cytokeratin-19 fragment • The precursor of surfactant protein B In the validation study of 63 ever-smoking patients with lung cancer and 90 matched controls, an integrated risk prediction model that combined smoking exposure with the biomarker panel score identified 40 of the 63 lung cancer cases, corresponding to a sensitivity of 0.63. By comparison, the US Preventive Services Task Force screening criteria demonstrated a sensitivity of 0.42 for these cases.
  • 20. Case Scenario 2 • 56 yrs old male came for master health check up, his chest X ray revealed right lung solitary pulmonary nodule • Patient has no symptoms • Not a Smoker • Systemic examination – no significant abnormality
  • 21. Case Scenario 2 • Solitary pulmonary nodule in chest X- ray. • Whether to observe or investigate further?
  • 22. Case Scenario 2 • Next plan of action to this patient? • Discharge the patient and ask him not to come unless and otherwise there are symptoms • Do a whole-body PET • Ask the radiologist the size of the nodule and decide further course of action • Refer the patient to the surgeon for lobectomy
  • 23. CT Chest • A 30 x 27 mm sized nodular mass lesion in the right middle lobe of 35 – 45 HU CT density in plain study and shows lobulated contour with multiple thin subpleural strips. • It shows few air bronchogram and feeding pulmonary vessel sign surrounded with focal areas of ground glass attenuation • 12 x 6 mm right paratracheal lymphnode and 4-8 mm sized aorto – pulmonary lymphnode
  • 24. Solitary pulmonary nodule malignancy risk in adults Herder Calculator Brock Calculator Answers calculated to formulate result: 1. Age? — 56 Years 2. Smoker (current or previous)? — No 3. Extra-thoracic cancer more than 5 years previous? — No 4. Diameter? — 30 mm 5. Upper Lobe? — No 6. Spiculated? — Yes 7. PET? — Moderate Results for Solitary Pulmonary Nodule Malignancy Risk Probability of Malignancy: 87.3 %
  • 26. Nodule Charecteristics • 30mm • Brock Model risk score 36.92% • Herder Model risk 87.3% • As per the algorithm the patient needs biopsy for finding out the definitive diagnosis • What is the diagnostic method of choice in this case?
  • 27. What is the most appropriate next step 1.Bronchoscopy 2.CT guided biopsy 3.Do a PET scan and decide 4.EBUS
  • 28. HPE image of the Biopsy • composed of goblet or columnar cells with abundant mucin
  • 29. Non-Small cell Lung Cancer • Non–small cell lung cancer (NSCLC) accounts for approximately 85% of all lung cancers • Histologically, NSCLC is divided into • adenocarcinoma, • squamous cell carcinoma (SCC) • large cell carcinoma • Often insidious, producing no symptoms • At initial diagnosis, • 20% of patients have localized disease • 25% of patients have regional metastasis • 55% of patients have distant spread of disease
  • 30. Etiology Exposure to carcinogens • Tobacco smoke contains more than 300 harmful substances with at least 40 known potent carcinogens. • Polyaromatic hydrocarbons and nicotine-derived nitrosamine ketone (NNK) are known to cause DNA damage by forming DNA adducts in animal models. • Benzo-A-pyrine also appears to induce molecular signaling such as AKT, as well as inducing mutations in p53 and other tumor suppressor genes. Exposure to Asbestos Exposure to Radon Outdoor air pollution Multiple Hit Theory
  • 31. Pathophysiology Genetic susceptibility • Amplification of oncogenes and inactivation of tumor suppressor genes in NSCLC ras family of oncogenes - H-ras, K-ras, and N-ras • Exclusively in adenocarcinoma and are found in 30% of such cases. • These mutations were not identified in adenocarcinomas that developed in persons who do not smoke. • The K-ras mutation appears to be an independent prognostic factor. Mutations in the oncogenes c-myc and c-raf APOBEC Tumor suppressor genes retinoblastoma (Rb) and p53.
  • 32. Lung Malignancy Adenocarcinoma • Arise from bronchial mucosal glands • Most Common • Non-Smokers • Peripheral location within the lung, • In some cases, at the site of pre-existing scars, wounds, or inflammation (Scar Ca) Squamous cell carcinoma • SCC accounts for 25-30% of all • Central parts of the lung - a cavitary lesion in a proximal bronchus • Characterized histologically by the presence of keratin pearls • Can be detected with cytologic studies, as it has a tendency to exfoliate. • It is the type most often associated with hypercalcemia Large-cell carcinoma • Large-cell carcinoma accounts for 10-15% • As a large peripheral mass • Sheets of highly atypical cells with focal necrosis - with no evidence of keratinization or gland formation.
  • 34. Symptoms due to Primary tumor • The symptoms produced by the primary tumor depend on its location (ie- central, peripheral). • Central tumors are generally squamous cell carcinomas (SCCs) and produce cough, dyspnea, atelectasis, postobstructive pneumonia, wheezing, and hemoptysis. • Most Peripheral tumors are adenocarcinomas or large cell carcinomas and, in addition to causing cough and dyspnea, can cause symptoms due to pleural effusion, and severe pain as a result of infiltration of parietal pleura and the chest wall. • Because of their peripheral location, adenocarcinomas may not call attention to themselves until they have produced extra thoracic metastases. For example, patients may present with clinical signs of bone spread or intracranial metastatic disease.
  • 35. Symptoms due to locoregional spread • Symptoms due to locoregional spread can include superior vena cava obstruction, • Paralysis of the recurrent laryngeal nerve, and phrenic nerve palsy, causing hoarseness and paralysis of the diaphragm; • Pressure on the sympathetic plexus, causing Horner syndrome; • Dysphagia resulting from esophageal compression; and pericardial effusion. • Superior sulcus tumors (Pancoast tumors) can cause compression of the brachial plexus roots as they exit the neural foramina, resulting in intense, radiating neuropathic pain in the ipsilateral upper extremity.
  • 36. Symptoms due to locoregional spread Pleural invasion / Mets may produce the following signs and symptoms: • Chest pain (27-49%) • Dyspnea (37-58%) • Cough (45-75%) Neurologic signs and symptoms include the following: • Arm weakness and paresthesias (brachial plexus impingement) • Miosis, ptosis, and anhidrosis (cervical sympathetic chain, Horner syndrome) • Dyspnea (secondary to phrenic nerve paralysis) Metastatic cancer may produce the following signs (8-68%): • Weight loss • Cachexia
  • 37. Symptoms due to distant spread Central nervous system (CNS) signs and symptoms include the following • Headache • Altered mental status • Seizure • Meningismus • Ataxia • Nausea and/or vomiting Vascular signs include the following: • Phlebitis • Thromboembolism (Trousseau syndrome) Musculoskeletal manifestations include the following • Bone pain (6-25%) • Spinal cord impingement
  • 38. Differential Diagnoses • Bacterial Pneumonia • Bronchitis • Mycoplasmal Pneumonia • Pleural Effusion • Pneumothorax • Tuberculosis (TB) • Viral Pneumonia • Pneumomediastinum • Pneumonia, Empyema, and abscess • Pneumothorax, tension and traumatic • Benign lung tumors • Carcinoid lung tumors • Granuloma • Hamartoma • Metastatic cancer • Small Cell Lung Cancer • Superior Vena Cava Syndrome in Emergency Medicine
  • 39. Diagnosis • After physical examination and CBC, chest x-ray is often the first test performed. • Chest radiographs may show the following: • Pulmonary nodule, • mass, or infiltrate • Mediastinal widening • Atelectasis • Hilar enlargement • Pleural effusion Methods of confirming diagnosis • Bronchoscopy • Sputum cytology • Mediastinoscopy • Thoracentesis • Thoracoscopy • Transthoracic needle biopsy (CT- or fluoroscopy-guided)
  • 40. Staging workup • Invasive staging procedures such as mediastinoscopy and mediastinotomy, Positron emission tomography (PET) scans may be useful in the detection of involved nodes, • The presence of which may influence decisions about operability • A combination of endosonography and surgical staging had a greater sensitivity for mediastinal nodal metastases than surgical staging alone
  • 41. Chest Radiography • Pulmonary nodule, mass, or infiltrate • Mediastinal widening • Atelectasis • Hilar enlargement • Pleural effusion
  • 42. Non–small cell lung cancer. Left upper collapse is almost always secondary to endobronchial bronchogenic carcinoma. Non–small cell lung cancer. Complete left lung collapse secondary to bronchogenic carcinoma of left mainstem bronchus. Chest Radiography
  • 43. Chest Radiography Non–small cell lung cancer. A cavitating right lower lobe squamous cell carcinoma. Non–small cell lung cancer. Right lower lobe opacity. Not well circumscribed and was found to be a squamous cell carcinoma
  • 44. Computed Tomography • Usually allow a presumptive differentiation between NSCLC and small cell lung cancer (SCLC). • Massive lymphadenopathy and direct mediastinal invasion are commonly associated with small cell carcinoma. • A mass in or adjacent to the hilum is a particular characteristic of SCLC
  • 45. Computed Tomography Lung cancer, small cell. Contrast-enhanced CT scan of the chest shows a large left lung and a hilar mass, with invasion of the left pulmonary artery. Non–small cell lung cancer. CT scan shows cavitation and air-fluid level
  • 46. Magnetic Resonance Imaging • MRI Spine - Spinal cord compression • MRI Brain - CNS metastasis • MRI Chest - Superior sulcus and Brachial plexus tumors
  • 47. Bone Scintigraphy • Lung cancer, small cell. Whole-body nuclear medicine bone scanning with anterior and posterior images reveal multiple abnormal areas of increased radiotracer activity in the pelvis, spine, ribs, and left scapula. • These findings are consistent with bony metastatic disease. • The bones are commonly affected in patients with small-cell lung cancer.
  • 48. Positron Emission Tomography • Lung cancer, small cell. Coronal positron emission tomogram shows abnormal areas of increased metabolic activity in the left hilar and left adrenal regions consistent with a hilar tumor with left adrenal metastasis
  • 49. Sputum Cytology • Centrally located endobronchial tumors may exfoliate malignant cells into sputum ( Esp Squamous Cell Ca) • False-positive rate for sputum cytology is 1%, • False-negative rate is as high as 40%. • Detects 71% of central tumors but less than 50% of peripheral tumors • Does not provide reliable distinction between different histologic subtypes
  • 50. Bronchoscopy • Direct visualization of the tumor • Determination of the extent of airway obstruction • Collection of diagnostic material under direct visualization • Direct biopsy of the visualized tumor • Bronchial brushings and washing, and transbronchial biopsies
  • 51. Biopsy • Transthoracic needle biopsy, guided by CT or fluoroscopy • false-negative rate is high at 26% • Diagnostic material can also be obtained from other abnormal sites (eg, enlarged palpable lymph nodes, liver, pleural or pericardial effusions, accessible bone lesions).
  • 52. Needle Thoracentesis • Both diagnostic and therapeutic in patients presenting with respiratory distress • sensitivity of only 80% with a specificity greater than 90%.
  • 53. Thoracoscopy • Thoracoscopy is usually reserved for tumors that remain undiagnosed after bronchoscopy or CT-guided biopsy. • Thoracoscopy is also an important tool in the management of malignant pleural effusions. • VATS - used to sample small peripheral tumors (≤2 cm), pleural tumors, or pleural effusions for diagnostic or staging purposes
  • 54. Radial Ebus for Peripheral Lung Nodules
  • 55.
  • 56. Molecular Testing • Evaluation of patients with metastatic non–small cell lung cancer (NSCLC). • EGFR & ALK mutation • EGFR inhibitors - erlotinib, gefitinib • ALK inhibitors - crizotinib, ceritinib • Cobas EGFR Mutation Test - EURTAC study • EGFR mutations (exon 19 deletions or exon 21 [L858R] substitution mutations • Ras mutation is a marker for aggressive disease and poor prognosis • Regimens to target this mutation - selumetinib plus docetaxel
  • 57. WHO classification of epithelial lung tumors Preinvasive lesions • Squamous dysplasia/carcinoma in situ • Atypical adenomatous hyperplasia • Diffuse idiopathic pulmonary neuroendocrine hyperplasia Invasive malignant lesions • Squamous cell carcinoma – Variants, papillary, clear cell, small cell, basaloid • Small cell carcinoma – Variant, combined small cell carcinoma • Adenocarcinoma • Acinar, • papillary, • bronchoalveolar, • Non mucinous (Clara cell/type II pneumocyte) type, • mixed mucinous and nonmucinous (Clara cell/type II pneumocyte and goblet cell) type or intermediate cell type, • solid adenocarcinoma with mucin formation, • adenocarcinoma with mixed subtypes, variants, • well-differentiated fetal adenocarcinoma, • mucinous (colloid) adenocarcinoma, • mucinous cystadenocarcinoma, • signet-ring adenocarcinoma, • clear cell adenocarcinoma
  • 58. WHO classification of epithelial lung tumors • Large cell carcinoma – Variants, • large cell neuroendocrine carcinoma, • combined large cell neuroendocrine carcinoma, • basaloid carcinoma, • lymphoepitheliomalike carcinoma, clear cell carcinoma, • large cell carcinoma with rhabdoid phenotype • Adenosquamous carcinoma • Carcinoma with sarcomatoid, • pleomorphic, or sarcomatous elements – • Carcinoma with spindle or giant cells, • pleomorphic carcinoma, • spindle cell carcinoma, • giant cell carcinoma, carcinosarcoma, • pulmonary blastoma • Carcinoid tumors – • Typical carcinoid, • atypical carcinoid • Carcinoma of salivary gland type – • Mucoepidermoid carcinoma, • adenoid cystic carcinoma, • others • Unclassified
  • 59. Staging • Chest CT scan is the standard for staging lung cancer • T describes the size of the primary tumor • N describes the spread of cancer to regional lymph nodes • M indicates whether the cancer has metastasized
  • 60. Staging - Primary tumor (T) T classification T components on CT Tis (AIS) Pure GGN ≤ 3 cm T1 T1mi ≤ 0.5 cm solid part within part-solid tumor total size ≤3 cm T1a 0.6–1.0 cm solid part within part-solid tumor total size ≤3 cm Pure GGN >3 cm ≤ 1 cm solid tumor T1b 1.1–2.0 cm solid part within part-solid tumor total size ≤3 cm >1–2 cm solid tumor T1c 2.1–3 cm solid part within part-solid tumor total size ≤3 cm >2–3 cm solid tumor T2 T2a 3.1–4 cm Involves main bronchus without involvement of carina T2b 4.1–5 cm Total/partial atelectasis Total/partial pneumonitis Involves hilar fat Involves visceral pleura (PL1 or PL2) T3 5.1–7 cm Separate tumor nodules in the same lobe as the primary Involves parietal pleura (PL3) Parietal pericardium Chest wall Phrenic nerve T4 >7 cm Involves diaphragm Mediastinal fat or other mediastinal structures (trachea, great vessels, heart, recurrent laryngeal nerve, esophagus) Carina Vertebral body Visceral pericardium Separate tumor nodules in the same lung but different lobes as the primary
  • 61. Staging - Lymph node (N) N classification N component on CT N0 No lymph node metastasis N1 Ipsilateral peripheral, intrapulmonary or hilar nodes metastasis N2 Ipsilateral mediastinal (upper, aortico- pulmonary, lower), subcarinal nodes metastasis N3 Ipsilateral or contralateral supraclavicular/scalene lymph node or contralateral mediastinal, hilar/interlobar, or peripheral nodes metastasis
  • 62. Staging - Metastatic (M) M classification M component on CT M0 No distal metastasis M1 M1a Intrathoracic metastasis Pleural effusion Pericardial effusion Contralateral lung nodules/pleural nodules M1b Single extrathoracic metastasis in a single organ M1c Multiple extrathoracic metastasis
  • 63. Stage grouping of the TNM system N0 N1 N2 N3 M0 Tis 0 T1mi IA1 T1a IA1 IIB IIIA IIIB T1b IA2 IIB IIIA IIIB T1c IA3 IIB IIIA IIIB T2a IB IIB IIIA IIIB T2b IIA IIB IIIA IIIB T3 IIB IIIA IIIB IIIC T4 IIIA IIIA IIIB IIIC M1a Tx IVA IVA IVA IVA M1b Tx IVA IVA IVA IVA M1c Tx IVB IVB IVB IVB
  • 64. Treatment • Approach Considerations • Surgery is the treatment of choice for patients with non–small cell lung cancer (NSCLC) stages I through IIIA. • In addition, patients with resected lung cancer have a high risk of relapse and so are treated with adjuvant chemotherapy. • Patients with stage IIIB and IV NSCLC are usually offered chemotherapy • Molecular-targeted therapy plays an increasingly important role in the treatment of advanced NSCLC. • Radiation is a reasonable option for treatment in patients who are not candidates for surgery. • The role of adjuvant radiation therapy after resection of the primary tumor remains controversial. • Palliative care concurrently with standard oncologic care at the initial diagnosis of advanced NSCLC
  • 65. Emergency Treatment • In cases of upper airway obstruction, admit the patient to the intensive care unit (ICU), prepare for intubation and/or cricothyrotomy, and obtain otolaryngologic and/or surgical consultation for fiberoptic laryngoscopy or intraoperative tracheostomy. • If hemoptysis is noted, administer supplemental oxygen and perform suctioning. If a threat of imminent demise exists, consider placing a double-lumen endotracheal tube. Position the patient with the bleeding hemithorax in a dependent position.
  • 66. Surgical Treatment • For all patients with stage I and II NSCLC—that is, those patients with no evidence of mediastinal disease or invasion of local organs • Lobectomy is the procedure of choice • Role of surgery for stage III disease is controversial • Patients with completely resectable primary tumors (ie, T4 N0) have a much better prognosis than those with spread to ipsilateral mediastinal or subcarinal lymph nodes (ie, N2), signifying that spread beyond the primary tumor is associated with a poor prognosis. • Patients with stage IIIB or IV tumors are almost never surgical candidates.
  • 67. Preoperative evaluation • Careful assessment of resectability, • Cardiopulmonary reserve, and perioperative risk. • High-resolution computed tomography (CT) and positron emission tomography (PET) scanning are helpful for preoperative planning in early- stage lung cancer. • Most patients with a preoperative forced expiratory volume in one second (FEV1) of greater than 2.5 L are able to tolerate pneumonectomy. • With an FEV1 of 1.1-2.4 L, a lobectomy is possible. • Patients with an FEV1 of less than 1 L are not considered candidates for surgery. • These factors are further modified by the presence of cardiac disease or other comorbid conditions.
  • 68. Postoperative evaluation and complications • Residual pulmonary function after surgical resection is estimated using pulmonary function tests and radionuclide lung scans • The perioperative mortality rate is 6% for pneumonectomy, 3% for lobectomy, and 1% for segmentectomy. These rates reflect improvements in anesthesia and surgical techniques.
  • 69. Radiation Therapy • In the treatment of stage I and stage II NSCLC, radiation therapy alone is considered only when surgical resection is not possible because of limited pulmonary reserve or the presence of comorbidities. • Radiation therapy alone as local therapy, in patients who are not surgical candidates, has been associated with 5-year cancer specific survival rates of 13-39% in early-stage NSCLC (ie, T1 and T2 disease) • Survival appears to be enhanced by the use of hyperfractionation schedules, such as continuous hyperfractionated accelerated radiotherapy (CHART) at 1.5 Gy 3 times a day for 12 days, as opposed to conventional radiation therapy at 60 Gy in 30 daily fractions. Overall survival at 4 years was 18% vs 12%. • Independent prognosticators of outcome for hyper fractionated radiation therapy • female sex, lower Karnofsky performance score (KPS), • less pronounced weight loss, squamous histology, lower stage, • shorter inter fraction interval, • Age did not influence overall survival or progression-free survival.
  • 70. Radiation Therapy • Beta blockers affect the metastatic tumors rather than the primary tumor. • Complex radiotherapy • Intermediate complexity radiotherapy • Stereotactic body radiotherapy (SBRT) • Radiofrequency ablation (RFA) • Adjuvant radiation therapy • Radiation therapy reduces local failures in completely resected (stages II and IIIA) NSCLC but has not been shown to improve overall survival rates.
  • 71. Systemic Chemotherapy • Approx 80% of all patients with lung cancer are considered for chemotherapy at some point during the course of their illness • At present, chemotherapy alone has no role in potentially curative therapy for NSCLC • Chemotherapy may be considered as part of multimodality therapy for locally advanced NSCLC and is used alone in the palliative treatment of stage IIIB NSCLC (owing to malig plef ) and stage IV NSCLC. • In advanced NSCLC, patients with good performance status ≥70% on the Karnofsky scale; and less than 10% body weight loss are good candidates for chemotherapy. • NSCLC is only moderately sensitive to chemotherapy, with single-agent response rates in the range of 15% or better. • Newer agents (eg, gemcitabine, pemetrexed, docetaxel, vinorelbine) have shown promising single-agent activity, with response rates from 20-25%.
  • 72. Platinum-based regimens • In younger patients, with a good performance status or in the adjuvant setting, cisplatin is preferred, but in older patients or those with significant comorbidities, carboplatin may be substituted. (ASCO) • In October 2012, the FDA approved protein-bound paclitaxel (Abraxane) for locally advanced or metastatic NSCLC, as first-line treatment in combination with carboplatin, in patients who are not candidates for curative surgery or radiation therapy • Cisplatin-gemcitabine did appear to have an increased progression free survival, compared with the standard treatment arm of cisplatin-paclitaxel (4.2 mo vs 3.4 mo), with increased renal toxicity Histologic factors in chemotherapy responsiveness • Cisplatin-pemetrexed is now the preferred combination for adenocarcinoma • squamous cell histology did better with the cisplatin – gemcitabine Genetic factors in resistance to platinum compounds • Excision repair cross-complementation group 1 (ERCC1) - high levels in tumor tissue have been associated with resistance to platinum. ( Significant in Male in comparision to Female ) • Increased expression of ribonucleotide reductase subunit 1 (RRM1) has been associated with decreased response to gemcitabine and platinum
  • 73. Second-line chemotherapy • Docetaxel and pemetrexed have been approved by the FDA in clinical setting of patients with good responses to first-line chemotherapy, good performance status, and a long disease-free period between initial chemotherapy and relapse • Dec 2014, the FDA approved ramucirumab (Cyramza) in combination with docetaxel for metastatic NSCLC with disease progression on or after platinum- based chemotherapy • Chemotherapy can give rise to various adverse effects, as follows: • Febrile neutropenia or bleeding from bone marrow suppression • Hyponatremia or hypomagnesemia from cisplatin nephrotoxicity • Renal failure or ototoxicity from cisplatin • Peripheral neuropathy from cisplatin, paclitaxel, and vinorelbine
  • 74. Combined Chemoradiation Therapy • Standard of care in the management of good-risk (ie, Karnofsky performance score of 70-100, minimal weight loss) patients with locally advanced unresectable (stage IIIA) non–small cell lung cancer (NSCLC) is combined-modality therapy consisting of platinum-based chemotherapy in conjunction with radiation therapy. • Chemotherapy regimens that have been studied in combination with radiation therapy include cisplatin/vinblastine and cisplatin/etoposide • In elderly patients or those with comorbidities and contraindications to cisplatin, weekly carboplatin/paclitaxel may be used • Consolidation therapy with the anti-programmed death ligand–1 antibody durvalumab following concomitant chemoradiation has shown significant benefit
  • 75. Molecular-Targeted Therapy • Molecular targeted therapies are revolutionized therapeutics which interfere with specific molecules to block cancer growth, progression, and metastasis. • Many molecular targeted therapies approved by the Food and Drug Administration (FDA), have demonstrated remarkable clinical success in the treatment of a myriad of cancer types including breast, leukemia, colorectal, lung, and ovarian cancers.
  • 76. Molecular-Targeted Therapy • All patients with NSCLC should have their tumor tissue tested for mutations, such as in the genes that code for endothelial growth factor receptor (EGFR), KRAS, anaplastic lymphoma kinase (ALK), ROS1, and programmed death ligand–1 (PDL1). • EGFR testing that identifies sensitivity to EGFR-directed tyrosine kinase inhibitors (TKIs) include exon 19 deletions or the L858R point mutation. The T790M and exon 20 insertion have been associated with low response or acquired resistance to TKIs. • In patients with adenocarcinomas that have EGFR mutations consistent with TKI sensitivity, options for single-agent targeted therapy without chemotherapy include the following • Osimertinib (Tagrisso) • Erlotinib (Tarceva) • Afatinib (Gilotrif) • Gefitinib (Iressa) • Dacomitinib (Vizimpro)
  • 77. Case Scenario 3 • A 70-year-old male came with complaints of right sided chest pain for the past 2 months , dyspnea for past 1 month ,swelling of face and upper limbs for the past 15 days. • chronic smoker – 50 pack years, chronic alcoholic. • Driver by occupation.
  • 78. • On examination - facial edema +,congested eyes+, b/l upper limbs swelling +, dilated veins seen over the chest wall and around the scapular areas. • Rs- decreased breath sound in right lower hemithorax, no added sounds.
  • 79.
  • 80. • Should we need to start treatment for this patient for SVC obstruction without HPE? or wait for HPE?
  • 81. Small cell Lung cancer (SCLC) • Small cell lung cancer (SCLC), previously known as oat cell carcinoma • Exhibits aggressive behavior, • rapid growth, • early spread to distant sites, • exquisite sensitivity to chemotherapy and radiation, and • association with distinct paraneoplastic syndromes, • hypercalcemia • Eaton-lambert syndrome • syndrome of inappropriate antidiuretic hormone (SIADH) secretion
  • 82. Pathophysiology • Small cell lung carcinoma (SCLC) arises in peribronchial locations and infiltrates the bronchial submucosa. • Widespread metastases occur early in the course of the disease, with common spread to the mediastinal lymph nodes, liver, bones, adrenal glands, and brain
  • 83. Prognosis • Extensive-stage SCLC Vs Limited-stage disease • Indicators of poor prognosis include the following: • Relapsed disease • Weight loss of greater than 10% of baseline body weight • Poor performance status • Hyponatremia • Elevated LDH level • Male sex • Elevated Alkaline Phosphatase level
  • 84. Complications • Hypercalcemia could initially be asymptomatic but in late stages could lead to weakness, fatigue, and sleepiness, and in extreme cases to severe constipation and lethargy. • Brain metastasis is often asymptomatic but could manifest as a unilateral eye abnormality, focal neurologic deficit, or at times with a new-onset headache that wakes the patient up. • Seizures
  • 85. Differential Diagnoses • Atypical Carcinoid Lung Tumour • Large Cell Neuroendocrine Carcinoma • Lung Adenoma • Lung Hamartoma • Mediastinal Lymphoma • Non-Small Cell Lung Cancer (NSCLC)
  • 86. Histology • Thought to develop from neuroendocrine Kulchitsky cells and are composed of sheets of small, round to spindled cells with dark nuclei, scant cytoplasm, and fine, granular (“salt and pepper”) nuclear chromatin with indistinct nucleoli. • Very high rates of cell division are observed, and necrosis, sometimes extensive, may be seen • Neurosecretory granules can be identified with the aid of electron microscopy. • The neuroendocrine nature of the neoplasm is suggested by its frequent association with neurologic and endocrine paraneoplastic syndromes • Immunohistochemical stains for chromogranin, neuron- specific enolase, CD56, and synaptophysin are usually positive
  • 87. Carcinoids SCLC LCNEC Synaptophysin, Chromogranin A, CD56/NCAM, TTF1 (50% are positive + staining weak and focal), Estrogen receptor (50% of carcinoids) * Synaptophysin, Chromogranin A (only weak), CD56 (most are positive), cytokeratins (AE1/AE3 or CAM 5.2), 34ßE12, TTF- 1 (90%) Synaptophysin, Chromogranin A (coexpressed in 70%), TTF-1 (50%) Common immunohistochemical markers
  • 88. Staging Overview • American Cancer Society (ACS) uses 2 types of staging—clinical and pathologic—for SCLC • Clinical staging involves physical examination, biopsy examinations, and imaging scans; • Majority of patients are staged with clinical staging, and this type of staging is usually used to describe SCLC tumor extent. • Pathologic staging is generally more accurate, as it includes clinical staging and adds postsurgical findings • Surgical findings may give the cancer a more advanced pathologic stage
  • 89. Staging systems VALSG 2-stage system (Veterans’ Administration Lung Study Group) • Limited-stage and Extensive-stage disease • Disease confined to one hemithorax, with or without involvement of the mediastinal, contralateral hilar or ipsilateral supraclavicular, or scalene lymph nodes are considered to have limited-stage disease, whereas those with disease involvement at any other location are considered to have extensive-stage disease • The key factor in defining limited-stage disease is the ability to encompass all of the disease within 1 tolerably safe radiation therapy port
  • 90. Staging systems TNM system • All solid tumors, including lung carcinomas, are staged using the tumor, node, metastasis (TNM) system, because it provides important prognostic information and is used to design management plans • TNM system fails to provide important prognostic information in patients with SCLC • AJCC – TNM • Limited-stage SCLC is defined as any T, any N, M0; the exception is T3-4, owing to multiple lung nodules that extend beyond a single radiation field • Extensive-stage disease as any T, any N, M1a/b, and T3-4, due to involvement of multiple lung nodules
  • 91. Treatment Approach Considerations • Clinical stage Ia (T1N0) after standard staging evaluation may be considered for surgical resection, but combined treatment with chemotherapy and radiation therapy is the standard of care. Radiation therapy is often added at the second cycle of chemotherapy • Patients with a good performance status and non bulky disease, intensive radiation therapy early in the course of treatment is indicated • Elderly patients with SCLC who have a good performance status and intact organ function should receive standard carboplatin-based chemotherapy • In 2018 - addition of atezolizumab—a humanized monoclonal anti– programmed death ligand 1 (PD-L1) antibody—to chemotherapy with carboplatin and etoposide (IMpower133 study )
  • 92. Limited-Stage SCLC - Standard Management • Combination chemotherapy and concurrent thoracic radiotherapy. • ACCP recommends four cycles of a platinum agent and etoposide • NCCN guidelines recommend a maximum of four to six cycles of cisplatin and etoposide • Radiotherapy - should begin as early as possible, preferably within 30 days of the start of chemotherapy • ACCP recommends concurrent accelerated hyper fractionated radiotherapy (twice-daily treatment) with platinum-based chemotherapy • NCCN guidelines recommend PCI (Prophylactic Cranial Irradiation) in patients with limited stage disease who have achieved a complete remission or in those with stage I disease who have undergone resection
  • 93. Extensive-Stage SCLC - Standard Management • Patients with extensive-stage small cell lung cancer (SCLC) are treated with combination chemotherapy alone. carboplatin or cisplatin plus etoposide has been the standard of care • Combination of chemotherapy with atezolizumab—a humanized monoclonal anti–programmed death ligand 1 (PD-L1) antibody—may represent a new standard of care. • Radiotherapy is used only to palliate symptoms, if required in extensive- stage SCLC • Gamma knife stereotactic radiosurgery • a salvage option for patients with brain metastases for whom previous whole-brain irradiation has failed
  • 94. Management of Relapsed SCLC • Patients with relapsed SCLC have an extremely poor prognosis. Individuals whose disease does not respond to or that progresses on initial treatment or those whose SCLC relapses within 6 months of completion of therapy have little chance of responding to additional chemotherapy • Administration of cisplatin and etoposide (PE) after vincristine (CAV) failure produces better response rates than does CAV given after PE Immune checkpoint modulation • In August 2018, the FDA granted accelerated approval of nivolumab for metastatic SCLC in patients with progression after platinum-based chemotherapy and at least 1 other line of therapy • FDA also granted accelerated approval for pembrolizumab in patients with metastatic SCLC with disease progression on or after platinum-based chemotherapy and at least 1 other prior line of therapy
  • 95. Management of Brain Metastases and Spinal Cord Compression • Management of symptomatic brain metastases includes high-dose corticosteroids and immediate whole brain radiation therapy. • In asymptomatic brain metastases, systemic chemotherapy may be initiated, with plans for close surveillance of the central nervous system (CNS) metastases and initiation of brain radiation after completion of systemic treatment. Spinal cord compression • An oncologic emergency • IV corticosteroids even before being sent for MRI • Radiation therapy and/or neurosurgical decompression, undertaken without delay.
  • 96. Management of Complications • Tumor lysis syndrome • Laboratory features of tumor lysis syndrome are hyperuricemia, hyperphosphatemia, hypocalcemia, and hyperkalemia • Patients should be well hydrated and, preferably, premedicated with allopurinol • Management of established tumor lysis syndrome is urinary alkalinization, correction of electrolyte abnormalities, and dialysis • Electrolyte abnormalities • Most common abnormality is hyponatremia, which, if severe, may cause neurologic symptoms and signs, including seizures, coma, and death • Other causes of hyponatremia (ie, volume depletion, abnormal renal function) must be excluded. • Fluid restriction and pharmacologic therapy in the form of demeclocycline
  • 97. SVC Syndrome Definition: The clinical signs and symptoms and radiographic manifestations resulting from obstruction of blood flow to the heart from the head and neck regions and upper extremities as a consequence of compression of the superior vena cava, either from direct invasion by the primary tumor into the mediastinum or from lymphatic spread with enlarged right paratracheal lymph nodes
  • 98. Signs of SVCS • Feeling of fullness in the head • Dyspnea • Cough • Dilated neck veins Prominent venous pattern on the face and the chest • Upper extremity and facial edema • Papilledema • Facial cyanosis • Plethora • Conjunctival edema
  • 99. Grades of SVCS • Grade 0 - SVC narrowing without clinical evidence of SVCS • Grade I - Moderate SVC narrowing without collateral • Grade II - Severe SVC narrowing with the azygos vein serving as partial collateral • Grade III - SVC obstruction above the azygos arch • Grade IV - SVC obstruction at or below the level of the azygos arch
  • 100. Collaterals in SVC Syndrome (a)Azygos-hemiazygos, (b)Paravertebral, (c)InternalMammary, (d)Anteriorjugular, (e)Thymic, (f)Lateralthoracic, (g)EsophagealVenous System (h)Thyroidal, (i)Pericardiophrenic Azygos and Hemiazygos Venous System Paravertebral Venous System
  • 101. Therapeutic Considerations • Goals of Therapy • Palliation of symptoms in the incurable situation • Aggressive management in the potentially curable situation. • Histologic diagnosis and staging are vital to determine whether palliation or cure is the goal • When the etiology is nonmalignant, relief of the SVC obstruction is the goal • Minimizing treatment-related morbidity is an important secondary goal.
  • 102. Symptomatic Relief • Head end elevation • Emergency radiotherapy • Glucocorticoids • Diuretics
  • 103. Head end elevation • Decreases hydrostatic pressure in upper body • Reduces head & neck edema • Not clinically proven
  • 104. Emergency radiotherapy • Not needed for most of patients • Obstruction is a chronic process. Emergency RT provides no benefit as long as the patients is clinically stable • May obscure diagnosis if biopsy is planned later
  • 105. Glucocorticoids • Laryngeal edema • Cerebral edema • Steroid responsive tumours - lymphoma / thymomas
  • 106. Diuretics • Commonly used • Not clinically proven
  • 107. Treatment of underlying cause • Stenting • Radiotherapy • Chemotherapy • Surgery
  • 108. Stenting • Rapid & sustained relief • Success rate 95-100 % • Under local anaesthesia • Per cutaneous procedure • Self expanding stents are preferred • May require more than one stents • Total occlusion is not a contraindication
  • 111. Surgical Reconstruction • Venous autografts produce the best results but are not always feasible • Pedicle created from the right atrium or the pericardium, • Aortic homografts • Synthetic grafts • Do not have an antigenicity problem • But are susceptible to intraluminal thrombosis • Patency rate of polytetrafluoroethylene grafts - 62%
  • 112. SURVIVAL • Mean survival for patients with malignancy ranged from 12.0 to 40.2 weeks • Median survival of all patients was 22 weeks • 5-year survival rate decreasing from 41% for lymphoma to 5% for small cell lung cancer to 2% for NSCLC
  • 113. Paraneoplastic Syndromes Definition • A paraneoplastic syndrome is a disease or symptom that is the consequence of the presence of cancer in the body, but is not due to the local presence of cancer cells. • Two of the most common are humoral hypercalcemia of malignancy (HHM) in squamous cell carcinoma and the syndrome of inappropriate antidiuretic hormone secretion (SIADH) in small cell lung cancer. • These phenomena are mediated by humoral factors (by hormones or cytokines)excreted by tumor cells or by an immune response against the tumor.
  • 114. Importance • Paraneoplastic syndromes occur in about 10% - 15% of malignant disease. • They may represent the earliest manifestation of an occult neoplasm. • They may mimic metastatic disease and therefore confound treatment. • They may serve as a TUMOR MARKER in previously treated patients to detect recurrence. • In patients undergoing ADJUVANT THERAPY to guide further treatment.
  • 116. Paraneoplastic endocrine syndromes associated with lung cancer Humoral hypercalcemia of malignancy Syndrome of inappropriate antidiuretic hormone production Cushing’s syndrome Hypoglycemia Acromegaly Carcinoid syndrome Gynecomastia Hyperthyroidism
  • 117. Criteria for the diagnosis of paraneoplastic endocrine syndromes • Abnormal endocrine function without physiologic feedback regulation • The absence of metastasis in the respective endocrine gland • Deterioration with increasing tumor burden • Improvement in endocrine function with the treatment of the tumor • Evidence of the presence of hormones in the tumor or hormone synthesis by the tumor
  • 118. Causes of hypercalcemia associated with lung cancer Humoral hypercalcemia of malignancy (1) Parathyroid hormone-related protein (2) Parathyroid hormone (3) 1,25-dihydroxyvitamin D (4) Granulocyte colony-stimulating factor Osteolytic activity at the sites of skeletal metastases
  • 119. Criteria for the diagnosis of Syndrome of inappropriate antidiuretic hormone Hyponatremia (serum sodium < 134 mEq/L) Hypoosmolality (plasma osmolality < 275 mOsm/kg) Inappropriately high urine osmolality ( > 500 mOsm/kg) Inappropriately high urinary sodium concentration ( > 20 mEq/L) Absence of hypothyroidism Absence of adrenal insufficiency Absence of volume depletion
  • 120. Treatment • Treatment of the underlying tumor. • Discontinue medications that contribute to hypercalcemia (eg, calcium supplements, vitamin D, thiazide diuretics, calcium-containing antacids, and lithium . • Fluid repletion with normal saline, which increases the glomerular filtration rate and inhibits renal calcium reabsorption. • IV bisphosphonates, Calcitonin • Mithramycin blocks bone resorption by inhibiting osteoclast RNA synthesis, Gallium nitrate-inhibition of osteoclastic bone resorption
  • 121. CONCLUSION • A paraneoplastic syndromes affect the presentation, clinical course, and treatment of cancer. • As a result of recent diagnostic and therapeutic advances, many paraneoplastic syndromes are currently well defined, have a clear pathogenesis, and have effective treatment options. • The ability to recognize and treat paraneoplastic syndromes may have a substantial effect on clinical outcomes, ranging from earlier cancer diagnosis, to improved quality of life, to increased delivery of tumor- directed therapy.
  • 122. Take Home points • High degree of suspicion important to diagnose Lung cancer at earlier stage • Screening for Lung cancer • Least invasive method should be employed to diagnose and stage the disease • At least appropriate treatment to alleviate the symptoms need to be taken care if no definite management possible.